100% found this document useful (1 vote)
763 views55 pages

Medical MCQs for Residents

The document contains a series of multiple choice questions about fluid, electrolyte, and acid-base disorders. Question 1 asks about the clinical scenario associated with hypercalcemia, with the answer being malignancy. Question 2 asks about calculating the sodium deficit in a patient with hyponatremia, with the answer being 700 mEq. Question 3 asks about the disturbance associated with tumor lysis syndrome, with the answer being hyperphosphatemia.

Uploaded by

Ashraf Shalbi
Copyright
© © All Rights Reserved
We take content rights seriously. If you suspect this is your content, claim it here.
Available Formats
Download as DOCX, PDF, TXT or read online on Scribd
100% found this document useful (1 vote)
763 views55 pages

Medical MCQs for Residents

The document contains a series of multiple choice questions about fluid, electrolyte, and acid-base disorders. Question 1 asks about the clinical scenario associated with hypercalcemia, with the answer being malignancy. Question 2 asks about calculating the sodium deficit in a patient with hyponatremia, with the answer being 700 mEq. Question 3 asks about the disturbance associated with tumor lysis syndrome, with the answer being hyperphosphatemia.

Uploaded by

Ashraf Shalbi
Copyright
© © All Rights Reserved
We take content rights seriously. If you suspect this is your content, claim it here.
Available Formats
Download as DOCX, PDF, TXT or read online on Scribd
You are on page 1/ 55

Resident MCQS

1. Which one of the following clinical scenarios is associated with hypercalcemia?

A. Fluid resuscitation from shock


B. Rapid infusion of blood products
C. Improper administration of phosphates
D. Malignancy
E. Acute pancreatitis

o Common causes include of hypocalcemia: infusion of large volumes of isotonic fluid,


administration of a citrate load, acute pancreatitis necrotizing fasciitis, renal failure,
gastrointestinal fistula, and hypoparathyroidism.

2. A 30-year-old, 70-kg woman has symptomatic hyponatremia. Her serum sodium level is 120
mEq/L (normal level, 140 mEq/L). Her sodium deficit is:

A. 500 mEq/L
B. 600 mEq/L
C. 700 mEq/L
D. 800 mEq/L
E. 400 mEq/L

o For the patient in question, the calculation is as follows: total body water = 70 kg × 0.5 = 35 L.
Sodium deficit = (140 − 120 mEq/L) × 35 L = 700 mEq sodium chloride.
o Rapid correction of hyponatremia can be associated with irreversible central nervous system
injury (central pontine and extrapontine myelinolysis)
o A correction of no more than 12 mEq/L/24 h should be achieved.

3. Which of the following disturbances is associated with tumor lysis syndrome?

A. Hypocalcemia
B. Hyporuricemia
C. Hypokalemia
D. Hypomagnesemia
E. Hypophosphatemia

 Hyperkalemia, hyperphosphatemia, hyperuricemia

4. An elderly patient with adult-onset diabetes mellitus is admitted to the hospital with severe
pneumonia. All of the following conditions can be associated with this patient condition except:

A. Hypokalemia
B. Hyperkalemia
C. Nonketotic hyperosmolar coma
D. Hypophosphatemia
E. Hyponatremia

5. An asymptomatic patient is found to have a serum calcium level of 13.5 mg/dL. Which of the
following medications should be avoided?

A. Bisphosphonates
B. Thiazide diuretics
C. Mithramycin
D. Calcitonin
E. Corticosteroids
6. Which of the following statements regarding the distribution, composition, and osmolarity of
body fluid compartments is true?

A. A majority of intracellular water resides in adipose tissue.


B. The principal intracellular anions are proteins and phosphates.
C. Sodium determines the effective osmotic pressure between the interstitial and intravascular
(plasma) fluid compartments.
D. Calcium greatly determines the effective osmotic pressure between the intracellular fluid
(ICF) and ECF compartments.
E. The principal intracellular cation is calcium.

o The major intracellular buffer consists of proteins and phosphate salts


o The principal intracellular anions are proteins and phosphates.
o The principal intracellular cations are potassium and magnesium
o The major extracellular buffer is bicarbonate.
o The principal extracellular anions are chloride and bicarbonate
o The principal extracellular cation is sodium
o Sodium greatly determines the effective osmotic pressure between the intracellular fluid
(ICF) and ECF compartments
o The non-permeable proteins in plasma are responsible for the effective osmotic pressure
between plasma and the interstitial fluid compartment (the colloid osmotic pressure).

7. Which of the following conditions is associated with hypernatremia?

A. Adrenal insufficiency
B. Tumor lysis syndrome
C. Marked hyperglycemia
D. Stevens–Johnson syndrome
E. Excessive loop diuretic administration

8. Which of the following humoral factors increases arterial vasodilation while not decreasing
protein permeability in the capillary membranes?

A. Bradykinin
B. Nitric oxide (NO)
C. Atrial natriuretic factor
D. Histamine
E. Platelet-activating factor

9. A 55-year-old female with a small bowel obstruction is found to have a serum potassium level of
2.8 mmol/L. Her hypokalemia is refractory to aggressive repletion. Which of the following is true?

A. The patient will likely suffer from flaccid paralysis and respiratory compromise until her
potassium level is increased to at least 3.0 mmol/L.
B. An electrocardiogram will likely show peaked T waves.
C. Intravenous potassium repletion with a rate of 80 mEq/h should improve her condition.
D. Hypomagnesemia could contribute to her problem.
E. Hypokalemia results in hypopolarization of the resting potential of the cell.

o An electrocardiogram may show depressed T waves and U waves


o Patients on digoxin are at increased risk for hypokalemia-associated arrhythmia
10. A 70-kg man is nil per os and receiving maintenance intravenous fluids in the form of 5%
dextrose in 0.45% saline after gastrointestinal surgery. Which of the following is true regarding his
fluid and electrolyte requirements?

A. His daily water need is 4000 mL.


B. His sodium requirement is 1 g/day.
C. His potassium requirement is 50 mEq/day.
D. Average urine volume is 3000 mL.
E. If he were febrile, his average increase in insensible loss would be 250 mL/day for each
degree of fever.

o Daily water need 2000 to 2500 mL per day


o Daily losses include 250 mL in stool, 800 to 1500 mL in urine, and approximately 600 mL as
insensible loss.
o In healthy individuals, 75% of insensible loss occurs through the skin and 25% through the
lungs.
o Sodium requirement is 2-3 mEq/K/day
o Potassium requirement is 1-2 mEq/K/day

11. Which of the following statements regarding hypervolemia in postoperative patients is true?

A. Hypervolemia can be produced by the administration of isotonic salt solutions in amounts


that exceed the loss of volume.
B. Acute overexpansion of the ECF space is typically not well tolerated in healthy individuals.
C. Excess administration of normal saline can result in metabolic derangement, most commonly
hyperchloremic metabolic alkalosis.
D. The most reliable sign of volume excess is peripheral edema.
E. Daily weight measurement in the postoperative period does not help determine fluid status.

o The earliest sign of volume excess during the postoperative period is weight gain.

12. With regard to potassium, which of the following statements is true?

A. Normal dietary intake of potassium is 150 to 200 mEq/day.


B. In patients with normal renal function, most ingested remains in the extracellular space.
C. More than 90% of the potassium in the body is located in the extracellular compartment.
D. Critical hyperkalemia (>6 mEq/L) is rarely encountered if renal function is normal.
E. Administration of sodium bicarbonate shifts potassium from the intracellular space (ICF) to
the extracellular space (ECF).

o The average daily dietary intake of potassium is 50 to 100 mEq


o Total extracellular potassium concentration is just 50 to 70 mEq (4.5 mEq/L),
o Total intracellular potassium concentration is just 150 mEq/L (more than 90% of the
potassium in the body)

13. Which of the following electrocardiographic (ECG) findings is associated with hyperkalemia?

A. Inverted T waves
B. Shortened PR interval
C. Peaked P waves
D. Narrowing of the QRS complex
E. T waves higher than R waves in more than one lead

o Peaked T waves and a prolonged PR interval, which are characteristic early ECG findings
14. With regard to postoperative hyponatremia, which of the following statements is true?

A. It does not occur when water is used to replace sodium containing fluids because
intracellular reserves often replace these losses.
B. In patients with head injury, hyponatremia despite adequate salt administration is usually
caused by occult renal dysfunction.
C. In oliguric patients, cellular catabolism with resultant metabolic acidosis increases cellular
release of water and can contribute to hyponatremia.
D. Hyperglycemia is not a cause of hyponatremia.
E. Patients with salt-wasting nephropathy usually have abnormal blood urea nitrogen and
creatinine values.

15. Which of the following statements regarding changes in volume status of the ECF compartment
is true?

A. Hyponatremia is diagnostic of excess ECF volume.


B. Hypernatremia is diagnostic of depletion of ECF volume
C. Excess extracellular volume is usually iatrogenic or due to renal or cardiac failure.
D. Central nervous system symptoms appear after tissue signs with acute volume loss.
E. The concentration of serum sodium is directly related to extracellular volume.

o The serum concentration of sodium is not necessarily related to the volume status of the ECF
compartment.
o Volume deficit is the most frequent volume disorder encountered during surgery.
o Central nervous system symptoms and cardiovascular symptoms appear after tissue signs
with acute volume loss.
o The tissue signs (e.g., decreased turgor, softness of the tongue with longitudinal wrinkling,
and atonicity of muscles) usually do not appear during the first 24 hours.
o Excess extracellular volume (volume overload) is usually iatrogenic or due to renal or cardiac
failure.

16. With regard to hypokalemia, which of the following statements is true?

A. It is less common than hyperkalemia in surgical patients.


B. Respiratory acidosis is associated with increased renal potassium loss.
C. Hypokalemia can cause increased deep tendon reflexes.
D. Flattened T waves and a prolonged QT interval are associated with hypokalemia.
E. Intravenous potassium administration should not exceed 10 to 20 mEq/h.

o Hypokalemia is more common than hyperkalemia in surgical patients.


o Respiratory and metabolic alkalosis result in increased renal potassium loss because
potassium is preferentially excreted in an attempt to preserve hydrogen ions.
o Signs of hypokalemia, including paralytic ileus, diminished or absent tendon reflexes,
weakness, and even flaccid paralysis, are related to decrease muscle contractility.
o As a rule, no more than 40 to 60 mEq of potassium should be added to each liter of
intravenous fluid, and the rate of potassium administration should never exceed 40 to 60
mEq/h.
17. With regard to abnormalities in serum sodium concentration, which of the following statements
is true?

A. Changes in serum sodium concentration usually produce changes in the status of ECF
volume.
B. The chloride ion is the main determinant of the osmolarity of the ECF space.
C. Extracellular hyponatremia leads to depletion of intracellular water.
D. Dry, sticky mucous membranes are characteristic of hyponatremia.
E. Preservation of normal ECF has higher precedence than does maintenance of normal
osmolality.

o Extracellular hyponatremia leads to extracellular osmolarity causes a shift of water into the
intracellular compartment.
o Water loss is the most common explanation for acute hypernatremia.
o Patients with central diabetes insipidus are treated with desmopressin (vasopressin) [DDAVP]
o Desmopressin is a synthetic analogue of ADH.

18. A 45-year-old alcoholic man is found to have hypomagnesemia. Which of the following
statements about magnesium is true?

A. The distribution of non-osseous magnesium is similar to that of sodium.


B. Calcium deficiency cannot be adequately corrected until the hypomagnesemia is addressed.
C. Magnesium depletion is characterized by depression of the neuromuscular and central
nervous systems.
D. Magnesium supplementation should be stopped as soon as the serum level has normalized.
E. The treatment of choice for magnesium deficiency is oral magnesium phosphate.

o The body contains 2000 mEq of magnesium, half of which is contained in bone.
o Most of the remaining magnesium is intracellular (a distribution similar to that of potassium).
o Hypomagnesemia like hypocalcemia is characterized by neuromuscular and central nervous
system hyperactivity.
o The extracellular magnesium concentration can be restored rapidly, but therapy must be
continued for 1 to 2 weeks to replenish the intracellular component.

19. Which of the following clinical situations can be associated with hypovolemic hyponatremia?

A. CHF
B. SIADH
C. Cirrhosis
D. Hyperglycemia
E. Gastrointestinal losses

Hyponatremia
Hypervolemic Euvolemic Hypovolemic
Excessive intake of hypotonic Hyperglycemia, Hyperlipidemia Decreased overall sodium
fluids or Hyperproteinemia intake or salt loss that
pseudohyponatremia exceeds water loss
Postoperative secretion of ADH SIADH Gastrointestinal losses,

Cirrhosis and CHF Diuretics Renal losses with diuretics


(High ECF volume) especially thiazide diuretics
water intoxication Primary renal disease
20. With regard to intraoperative management of fluids, which of the following statements is true?

A. In a healthy person, up to 500 mL of blood loss may be well tolerated without the need for
blood replacement.
B. During an operation, functional ECF volume is directly related to the volume lost to suction.
C. Functional ECF losses should be replaced with plasma.
D. Administration of albumin plays an important role in the replacement of functional ECF
volume loss.
E. Operative blood loss is usually overestimated by the surgeon.

21. Which of the following statements regarding total body water is true?

A. In males, approximately 40% of total body weight is water.


B. The percentage of total body weight that is water is higher in females than in males.
C. Obese individuals have a greater proportion of water (relative to body weight) than lean
individuals.
D. The percentage of total body water decreases with age.
E. The majority of body water is contained within the interstitial fluid compartments.

o In males, approximately 60% of total body weight is water.


o In females, approximately 50% of total body weight is water.
o Fat contains little water
o Lean individuals have a greater proportion of water (relative to body weight) than obese
individuals
o Total body water decreases with age as a result of decreasing lean muscle mass.
o In infants, approximately 60% of total body weight is water.
o ICF compartment (40% of body weight)
o ECF compartment (20% of body weight),
o Interstitial compartment (15% of body weight)
o Intravascular fluid compartments (5% of body weight)

For the ‘average’ 70 Kg man

o Total body water is 42 L (~60% of body weight)


o 28 L is in the intracellular and 14 L in the extracellular compartments
o The plasma volume is 3 L
o The extravascular volume is 11 L
o Total body Na+ is 4200 mmol  (50% in ECF)
o Total body K+ is 3500 mmol  (only about 50-60 mmol in ECF)
o Normal osmolality of ECF is 280 –295 mosmol/kg

22. A 62-year-old female takes 40 mg of furosemide twice daily for hypertension and CHF. Which of
the following is true?

A. Loop diuretics act on the distal convoluted tubule in the nephron.


B. Magnesium is affected by loop diuretics.
C. Fatigue and muscle weakness are not side effects of her medication.
D. Loop diuretics decrease venous capacitance.
E. Loop diuretics are agonists to the sodium-potassium chloride cotransporter.

o Loop diuretics, such as furosemide, are potent inhibitors of the sodium-potassium-chloride


cotransporter.
o Loop diuretics act on the thick ascending limb of the loop of Henle in the nephron.
o Loop diuretics inhibition of sodium absorption, they increase blood flow to the kidneys by
stimulating vasodilatory prostaglandins and increase venous capacitance
23. With regard to distributional shifts during an operation, which of the following statements is
true?

A. The surface area of the peritoneum is not large enough to account for significant third-space
loss.
B. Approximately 1 to 1.5 L/h of fluid is needed during an operation.
C. Blood is replaced as it is lost, without modification of the basal operative fluid replacement
rate.
D. Sequestered ECF is predominantly hypotonic.
E. A major stimulus to ECF expansion is peripheral vasoconstriction.

o The functional ECF volume decreases during major abdominal operations due to
1. Extensive dissection,
2. Fluid collection within the lumen and wall of the small bowel, and
3. Accumulation of fluid in the peritoneal cavity.
o The surface area of the peritoneum is 1.8 m2
o Sequestered ECF is predominantly isotonic.
o Blood is replaced as it is lost, regardless of additional fluid therapy
o Lost ECF should be replaced during the operative procedure
o Approximately 0.5 to 1.0 L/h of fluid is needed during an operation

24. With regard to perioperative fluid management, which of the following statements is correct?

A. Insensible loss is approximately 600 mL/day.


B. Intraoperative insensible losses from an open abdomen are less than 250 mL/h.
C. About 200 to 300 mL of fluid is needed to excrete the catabolic end products of metabolism.
D. Lost urine should be replaced milliliter for milliliter.
E. Hypermetabolism and hyperventilation are not important factors in postoperative fluid loss
or management.

o Approximately 800 to 1000 mL/day of fluid is needed to excrete the catabolic end products
of metabolism.

25. Which of the following has no effect on the development of hypernatremia?

A. Excessive sweating
B. Hyperlipidemia
C. Lactulose
D. Glycosuria
E. Inadequate maintenance fluids

Hypernatremia
Hypovolemic Euvolemic Hypervolemic
Decrease intake of fluids Pure water loss Sodium gain
Hypotonic water loss
Urine sodium<20 mmol/L Renal loss: central diabetes Sodium bicarbonate infusion
GI loss insipidus Hypertonic diaylasis
(diarrhea/NG secretion)
Burn or excess sweeting
Urine sodium>20 mmol/L Extra-renal loss: insensible loss Primary aldosteronism
Loop diuretic Congenital adrenal
Osmotic diuresis hyperplasia.
(mannitol/ glycosuria/ post
obstruction)
Cushing syndrom
26. With regard to diabetes insipidus, which of the following statements is true?

A. Diabetes insipidus causes hypervolemic hyponatremia.


B. Central diabetes insipidus cannot be corrected by the administration of desmopressin.
C. Treatment of diabetes insipidus requires correction of hypernatremia at a rate faster than 12
mEq/day.
D. Alcohol intoxication can mimic diabetes insipidus by suppression of vasopressin
E. Lithium administration could induce central diabetes insipidus.

o Central diabetes insipidus (lack of production of ADH by the hypothalamus)


o Nephrogenic diabetes insipidus (lack of response of the distal tubule of the nephron to
ADH).
o Too rapid correction of hypernatremia may lead to cerebral edema and seizures
o Desmopressin is a synthetic analogue of ADH
o Central diabetes insipidus will respond to desmopressin, whereas nephrogenic diabetes
insipidus will not.
o Lithium and amphotericin B can induce nephrogenic, not central diabetes insipidus.

27. A postoperative patient has a serum sodium concentration of 125 mEq/L and a blood glucose
level of 500 mg/dL (normal level, 100 mg/dL). What would the patient’s serum sodium
concentration be (assuming normal renal function and appropriate intraoperative fluid therapy) if
the blood glucose level were normal?

A. 120 mEq/L
B. 122 mEq/L
C. 137 mEq/L
D. 142 mEq/L
E. 147 mEq/L

o Serum osmolality = 2x NA + (Glucose/18) + (BUN/2.8)


o As a general rule, each 100-mg/dL rise in the blood glucose level above normal is equivalent
to a 1.6- to 3.0-mEq/L fall in the apparent serum sodium concentration.

28. Which one of the following is least useful in the immediate treatment of hyperkalemia?

A. Calcium salts
B. Sodium bicarbonate
C. Potassium-binding resins
D. Glucose and insulin
E. Hemodialysis

29. Which one of the following is not associated with hypocalcemia?

A. Shortening of the QT interval


B. Painful muscle spasms
C. Perioral or fingertip tingling
D. Seizures in children
E. Prolongation of the QT interval

o Calcium is found in three forms in the body:


1. Protein bound (≈50%, mostly to albumin)
2. Diffusible calcium combined with anions such as bicarbonate, phosphate, and acetate
(5%)
3. Ionized (≈45%)

o The symptoms of hypocalcemia are generally seen at serum levels of less than 8 mg/dL.
o Symptoms include numbness and tingling in the circumoral area and in the tips of the fingers
and toes.
o Signs include hyperactive deep tendon reflexes, positive Chvostek sign, positive Trousseau
sign, muscle and abdominal cramps, tetany with carpal pedal spasm, or convulsions.

30. Which one of the following clinical signs or symptoms is associated with serum sodium
concentrations below 125 mEq/L?

A. Restlessness
B. Hallucinations
C. Tachycardia
D. Hyperventilation
E. Hyperthermia

31. Which one of the following clinical signs or symptoms is not associated with serum sodium
concentrations below 125 mEq/L?

A. Headache
B. Hallucinations
C. Bradycardia
D. Hypoventilation
E. Hyperthermia

o Also hypothermia, seizures, lethargy.

32. Which one of the following is not a stimulus for ECF expansion?

A. Hemorrhage leading to a reduction in blood volume


B. Increased capillary permeability after major surgery
C. Peripheral arterial vasoconstriction
D. Negative interstitial fluid hydrostatic pressure
E. Colloid oncotic pressure

o Approximately 85% of the ECF that is within the vascular compartment resides in the venous
circulation. Therefore, the remaining 15% resides within the arterial system.
o Serum albumin is a major determinant of colloid oncotic pressure
o Accounts for 60-80% of normal plasma oncotic pressure

33. A 70-year-old man with sepsis has a pH of 7.18. Which of the following statements is true
regarding his metabolic acidosis?

A. Tissue hypoxia leads to increased oxidative metabolism.


B. Acute compensation for metabolic acidosis is primarily renal.
C. Metabolic acidosis results from the loss of bicarbonate or the gain of fixed acids.
D. The most common cause of excess acid is prolonged nasogastric suction.
E. Restoration of blood pressure with vasopressors corrects the metabolic acidosis associated
with circulatory failure.

34. A 70-kg man with pyloric obstruction secondary to ulcer disease is admitted to the hospital for
resuscitation after 1 week of prolonged vomiting. What metabolic disturbance is expected?

A. Hypokalemic, hyperchloremic metabolic acidosis


B. Hyperkalemic, hypochloremic metabolic alkalosis
C. Hyperkalemic, hyperchloremic metabolic acidosis
D. Hypokalemic, hypochloremic metabolic alkalosis
E. None of the above
35. Which of the following statements regarding total body water is false?

A. In males, approximately 60% of total body weight is water


B. The percentage of total body weight that is water is higher in males than in females
C. Lean individuals have a greater proportion of water (relative to body weight) than do obese
individuals
D. The percentage of total body water decreases with age
E. The majority of body water is contained within the interstitial fluid compartment

36. Which of the following statements regarding the distribution, composition, and osmolarity of
body fluid compartments is not true?

A. Most intracellular water resides in skeletal muscle.


B. The principal extracellular cation is sodium.
C. The principal extracellular anions are chloride and bicarbonate.
D. Non-permeable proteins determine the effective osmotic pressure between the interstitial
and intravascular (plasma) fluid compartments.
E. Calcium greatly determines the effective osmotic pressure between the ICF and ECF
compartments

37. Which of the following statements regarding hypervolemia in postoperative patients is not
true?

A. Hypervolemia can be produced by the administration of isotonic salt solutions in amounts


that exceed the loss of volume.
B. Acute overexpansion of the ECF space is usually well tolerated in healthy individuals.
C. Avoidance of volume excess requires daily monitoring of intake and output and
determinations of serum sodium concentrations to guide accurate fluid administration.
D. The most reliable sign of volume excess is peripheral edema.
E. The earliest sign of volume excess is weight gain

38. Which of the following statements regarding loop diuretics is not true?

A. Loop diuretics act on the thick ascending limb of the loop of Henle in the nephron.
B. Loop diuretics increase blood flow to the kidney.
C. Magnesium and calcium are unaffected during diuresis.
D. Loop diuretics increase venous capacitance.
E. Loop diuretics inhibit the sodium-potassium-chloride cotransporter.

39. Which of the following pairing statements regarding daily fluid balance is incorrect?

A. Daily water intake, 2000 to 2500 mL


B. Average stool loss, 1000 mL
C. Average insensible loss, 600 mL
D. Average urine volume, 800 to 1500 mL
E. Average increase in insensible loss in a febrile patient, 250 mL/day for each degree of fever

40. Which of the following statements concerning the sodium concentration of various fluids is
incorrect?

A. Pancreatic secretions, 140 mEq/L


B. Sweat, 40 mEq/L
C. Gastric secretions, 50 mEq/L
D. Saliva, 100 mEq/L
E. Ileostomy output, 125 mEq/L
o Pancreatic secretions have high bicarbonate concentrations (75 mEq/dL)
o Stomach, small intestine, and biliary fluids have relatively high chloride concentrations.
o Duodenal, ileal, pancreatic, and biliary fluids contain levels of sodium that approximate those
seen in plasma.
o Saliva is relatively high in potassium

Sodium chloride potassiu calcium lactate


m
Lactated Ringer solution 130 mEq/L 109 mEq/L 4 mEq/L 3 mEq/L 28 mEq/L
0.9% normal saline solution 154 mEq/L 154 mEq/L
Hypertonic 3% saline solution 513 mEq/L 513 mEq/L

41. With regard to postoperative fluid management, which of the following statements is not true?

A. Insensible loss is approximately 600 mL/day.


B. Insensible loss may increase to 1500 mL/day.
C. About 800 to 1000 mL of fluid is needed to excrete the catabolic end products of
metabolism.
D. Lost urine should be replaced milliliter for milliliter.
E. Lost gastrointestinal fluids should be replaced milliliter for milliliter.

42. Which of the following conditions is not associated with hypernatremia?

A. Diabetes insipidus
B. Tumor lysis syndrome
C. Steven-Johnson syndrome
D. Primary hypodipsia
E. Enterocutaneous fistula

43. With regard to postoperative hyponatremia, which of the following statements is not true?

A. It may easily occur when water is used to replace sodium containing fluids or when the water
given exceeds the water lost.
B. In patients with head injury, hyponatremia despite adequate salt administration is usually
caused by occult renal dysfunction.
C. In oliguric patients, cellular catabolism with resultant metabolic acidosis increases cellular
release of water and can contribute to hyponatremia.
D. Hyperglycemia may be a cause of hyponatremia.
E. Patients with salt-wasting nephropathy could have normal blood urea nitrogen and
creatinine values.

44. With regard to potassium, which of the following statements is not true?

A. Normal dietary intake of potassium is 50 to 100 mEq/day.


B. In patients with normal renal function, most ingested potassium is excreted in urine.
C. More than 90% of the potassium in the body is located in the extracellular compartment.
D. Critical hyperkalemia (>6 mEq/L) is rarely encountered if renal function is normal.
E. Administration of sodium bicarbonate shifts potassium from the extracellular space (ECF) to
the intracellular space (ICF).

45. Which of the following electrocardiographic (ECG) findings is not associated with hyperkalemia?

A. Peaked T waves
B. Prolonged PR interval
C. Loss of the P wave
D. Narrowing of the QRS complex
E. T waves higher than R waves in more than one lead
46. Which of the following disturbances is not associated with tumor lysis syndrome?

A. Hypocalcemia
B. Hyperuricemia
C. Hyperkalemia
D. Hypermagnesemia
E. Hyperphosphatemia

o Hyperkalemia, hyperphosphatemia, hyperuricemia, Hypocalcemia

47. Apnea develops in a postoperative patient from narcotics. His Pco2 is 60. With regard to acid-
base buffering, which of the following is not true?

A. The major extracellular buffer is bicarbonate.


B. Intracellular pH and extracellular pH are usually the same.
C. The major intracellular buffer consists of proteins and phosphate salts.
D. Hydrogen ions cannot directly pass through the cell membrane.
E. Treating acidosis with bicarbonate infusion can cause cell death.

o Intracellular pH is maintained at 7.1, whereas extracellular pH is normally 7.4.

48. For an adult patient consuming a normal diet, which of the following is the most calorically
dense energy source?

A. Fat
B. Alcohol
C. Protein
D. Carbohydrate
E. Water

49. A 53-year-old diabetic patient undergoes small bowel resection for volvulus. He now has a
prolonged postoperative ileus and has had only 0.45% normal saline for 5 days. In order to limit
protein catabolism, how much glucose should be administered in his total parenteral nutrition
(TPN)?

A. 1 L of 5% dextrose in water (D5W)


B. 2 L of D5W
C. 3 L of D5W
D. 4 L of D5W
E. 5 L of D5W

o Glucose tolerance is determined by the rate at which mechanisms of glucose removal can
operate.
o Administration of 100 g of glucose (or 1 mg/ kg/min) has a protein-sparing effect that
suppresses the use of nitrogen (from amino acids) for gluconeogenesis.
o D5W, or 5% dextrose per liter of water, contains 50 g of dextrose per liter.

50. A patient with a history of a trauma laparotomy presents with a small bowel obstruction. A
nasogastric tube is placed that aspirates greater than 4L of fluid per day. The gastrointestinal tract
can secrete and reabsorb how much water in the form of gastric juices per day (in a 70-kg adult
male)?

A. 1 to 2 L/day
B. 4 to 5 L/day
C. 6 to 7 L/day
D. 8 to 10 L/day
E. 50 L/day
51. In the above patient, the decreased insulin–glucagon ratio seen during simple starvation allows:

A. Increased lipogenesis
B. Increased lipolysis
C. Increased protein synthesis
D. Increased glycogen production
E. Decreased lipolysis

o During starvation, the insulin–glucagon ratio is decreased, which allows activation of lipolysis
and suppression of lipogenesis.
o Lipolysis breaks down fat to free fatty acids
o Many tissues prefer fat as a fuel source (i.e., kidney, cardiac muscle, and skeletal muscle).
o The liver oxidizes fatty acids to acetyl CoA, which is then converted to ketones

52. During simple starvation, gluconeogenesis is important for:

A. Glycogen storage
B. Lipogenesis to continue to allow adequate fat storage
C. Protein synthesis to progress to allow muscle health
D. Tissues that use only glucose for fuel, such as the brain and blood, and are dependent on this
process
E. None of the above

o In early fasting, glycogenolysis is the first energy source, followed by gluconeogenesis to


maintain a glucose supply to obligate glucose users such as the brain, blood, renal medulla,
and bone marrow.
o Gluconeogenesis requires alanine, glycerol, and lactate

53. Which amino acid is released in large amounts to be used by the liver during simple starvation?

A. Valine
B. Serine
C. Glutamine
D. Cysteine
E. Homocysteine

54. A 24-year-old male is made nil per os (NPO) after midnight for lipoma removal the next day. His
case is delayed until early evening, and he enters simple starvation. In this patient, glucagon
mobilizes which of the following?

A. Glycogen from muscle tissue


B. Liver glycogen
C. Insulin to improve the cellular uptake of glucose
D. Glucose to the liver for storage
E. None of the above

55. A patient presents with lymphoma and a left pleural effusion; upon drainage, the fluid is noted
to be chylous. In which of the following is he likely to be deficient?

A. Short-chain fatty acids


B. Medium-chain fatty acids
C. Long-chain fatty acids
D. Calcium
E. Vitamin B12

o Chylothorax and other lymphatic or thoracic duct disruptions can result in a profound
deficiency of TAGs and long-chain fatty acids.
56. A patient presents with a 55% total body surface area burn. Early enteral feeds are started per
protocol, and the burn unit director considers glutamine supplementation. Glutamine is an amino
acid that:

A. Is categorized as an essential amino acid


B. Is found only in muscle tissue
C. Has been shown to be conditionally essential during stress
D. Maintains stable levels in plasma during stress
E. Can be eliminated from the diet during times of stress

Glutamine
o Is the most abundant amino acid in the body,
o Is a conditionally essential amino acid;
o It accounts for 50% of the amino acids in muscle
o Its concentrations can fall during times of stress because of the body’s inability to meet
increases in body requirements for the amino acid
o Is the major amino acid for intestinal mucosa,

57. Which amino acids are classified as essential, can be metabolized outside the liver, and are a
local source of energy for muscle?

A. Leucine, isoleucine, and valine


B. Alanine, arginine, and lysine
C. Ethionine, glutamine, and lysine
D. Phenylalanine, tyrosine, and histidine
E. None of the above

58. A 65-year-old woman with recently diagnosed non-obstructing colon cancer presents to
schedule a right hemicolectomy. She has read about nutrition and healing after her surgery and
asks how she can optimize her protein intake. What are the dietary protein recommendations for a
60-kg woman with intact protein stores?

A. 0.7 to 0.8 g/kg/day (42 to 48 g/day)


B. 0.9 to 1.0 g/kg/day (54 to 60 g/day)
C. 1.2 to 1.5 g/kg/day (72 to 90 g/day)
D. 2 to 4 g/kg/day (120 to 240 g/day)
E. 5 to 6 g/kg/day (300 to 360 g/day)

o The dietary protein requirement for adults is 0.8 g/ kg/day; that is, approximately 20% of the
calories consumed should be in the form of protein.
o One gram of nitrogen equals 6.24 g of protein.

59. A patient presents with a perforated Meckel’s diverticulum and undergoes terminal ileal
resection with re-anastomosis. He subsequently develops poor reabsorption of bile acids. What is
the primary substrate for the formation of bile acids?

A. Cholesterol
B. Triglycerol
C. Triglycerides
D. Phospholipids
E. Insulin

o Three main forms of fat are found in the body: glycerides, phospholipids, and sterols.
o Sterols Consist primarily of cholesterol
o The primary bile acids are cholate and chenodeoxycholate
60. Which of the following forms of fat constitute 95% to 98% of fat in the body?

A. Glycerides
B. Phospholipids
C. Sterols
D. Cholesterol
E. Linoleic acid

Glycerides
o principally triglycerides and triglycerol (fatty acid and glycerol),
o Are the storage forms of fat
o Are the most abundant forms in food; they account for approximately 95%–98% of ingested
fat and the fat in tissues.
o Triglycerides store calories, protect organs, and act as insulators.

61. A 26-year-old patient presents with abdominal pain, weight loss, and steatorrhea. He is found
to have severe terminal ileal involvement and structuring. In which of the following vitamins is the
least likely to be deficient?

A. Vitamin A
B. Vitamin D
C. Vitamin E
D. Vitamin C
E. Vitamin K

o Vitamin A deficiency —dermatitis, night blindness, and poor wound healing


o Vitamin D deficiency —bone demineralization and osteopenia
o Vitamin E deficiency —increased platelet aggregation and decreased red blood cell survival
o Vitamin K deficiency —bruising and hemorrhage

62. Which of the following is not a component of the Harris– Benedict equation?

A. Weight
B. Height
C. Age
D. Gender
E. Lean body mass %

o In general, a patient’s diet should be broken into 20% protein, 30% fat, and 50%
carbohydrates.

63. A 75-year-old female presents with ductal carcinoma in situ for simple mastectomy. She has
marked muscle wasting on examination and admits to a poor diet. Which of the following values is
most predictive of postoperative mortality?

A. Serum sodium
B. Serum albumin
C. Serum protein
D. Serum creatinine
E. Serum glucose
64. A 55-year-old woman presents with pneumoperitoneum and is found to have a perforated
gastric ulcer with gross intraabdominal spillage. Subsequently, she has sepsis and a prolonged
intubation, and tube feeds are started. Which of the following visceral proteins is the best indicator
of her immediate nutritional status?

A. Pre-albumin
B. Albumin
C. Transferrin
D. Total protein
E. Serum globulin

Serum albumin Half-life of 18 to 21 days


Serum transferrin Half-life of 8 to 10 days
Serum pre-albumin Half-life of 10 h to 2 days

65. Which of the following visceral proteins has the shortest half-life?

A. Retinol-binding pre- albumin


B. Albumin
C. Transferrin
D. Thyroxine-binding pre- albumin
E. Serum globulin

66. A patient presents to the trauma bay after a house fire with a 65% TBSA burn. He is intubated
and remains in the burn unit for 5 days, after which he enters the hypermetabolic flow phase of
stress metabolism. Hyperglycemia during stress hypermetabolism can be attributed to:

A. Increased insulin resistance


B. Increased glycogen storage
C. Decreased lipolysis
D. Increased glycogenesis
E. Increased insulin uptake

67. In the patient above, stress hypermetabolism is characterized by:

A. Decreased body temperature


B. Hypoglycemia and glycogenesis
C. Fluid imbalance and increased resting metabolic rate
D. Decreased gluconeogenesis and proteolysis
E. Decreased urinary protein retention

o The earliest stages of hypermetabolism are characterized by


1. Increases in gluconeogenesis,
2. Increases resting energy expenditure (REE),
3. Proteolysis,
4. Ureagenesis, and
5. Urinary nitrogen loss.
o which are stimulated by endocrine (hormonal) changes:
1. Increased cortisol,
2. Increased glucagon,
3. Increased catecholamines,
4. Decreased insulin.
68. In diabetic patients or those in a fasting state, lipolysis can exceed carbohydrate breakdown
and:

A. Elevate insulin utilization


B. Increase the production of fatty acids, which are then converted to ketones
C. Decrease lipase utilization
D. Decrease acetoacetate production
E. Improve a patient’s response to medical therapies

69. A 59-year-old woman remains intubated due to sepsis after perforated diverticulitis. She has
failed multiple trials of extubation due to hypercapnia, which you suspect to be due to overfeeding.
Which of the following RQs is characteristic of overfeeding?

A. 0.75
B. 0.85
C. 0.90
D. 1.0
E. 1.05

70. A 35-year-old man is admitted to the intensive care unit (ICU) following a diagnosis of acute
pancreatitis. After initial resuscitation, the patient’s condition improves and enteral tube feedings
are started through a postpyloric tube. Initial intolerance to a tube feeding regimen requires the
clinician to:

A. Immediately start TPN


B. Add water to the feeding regimen to dilute the feedings for better tolerance
C. Slow the tube feeding regimen and progress to the goal rate less aggressively
D. Immediately change the tube feeding formula
E. Increase the tube feeding rate per hour

71. A 67-year-old woman with a history of atrial fibrillation is admitted to the emergency
department with complaints of abdominal pain out of proportion to the physical findings. She is
found to have superior mesenteric arterial occlusion and undergoes on-table thrombectomy and
stenting with small bowel resection. The remaining proximal jejunum measures 100 cm. What is the
minimum amount of small intestine required for absorption of nutrients before considering the use
of enteral feedings?

A. 20 cm of small intestine
B. 50 cm of small intestine
C. 100 cm of small intestine
D. 120 cm of small intestine
E. 250 cm of small intestine

72. Due to multiple takebacks for “second-look” procedures and attempts at extubation, the patient
in the question above remains without enteral nutrition for 8 days. Which type of formula might be
appropriate for a patient who has been NPO for more than 1 week and has a partially functioning
gastrointestinal tract?

A. Elemental formula
B. Concentrated formula
C. Specialty formula
D. Modular formula
E. Superconcentrated formula

o Elemental formula might be appropriate for a patient who has had nothing per mouth for
more than a week and has a partially functioning gastrointestinal tract?
73. Which of the following is one of the most common food allergies that must be considered when
deciding on a tube feeding formula?

A. Rice allergy
B. Soy allergy
C. Nut allergy
D. Corn syrup allergy
E. Citrus fruit allergy

74. How many calories are provided in one 500-mL bottle of 20% intravenous fat solution?

A. 150 kcal
B. 550 kcal
C. 800 kcal
D. 1000 kcal
E. 4000 kcal

o In this question, 500 mL of a 20% infusion provides 1000 kcal (2.0 kcal/mL x 500 = 1000 kcal)

75. A 65-year-old man is admitted to the hospital because of profuse diarrhea after small bowel
resection for an ischemic bowel that resulted in short bowel syndrome. The patient is resuscitated
and TPN started. What is the maximum infusion rate for lipids when using TPN?

A. 0.5 g/kg/day
B. 1.5 g/kg/day
C. 2.5 g/kg/day
D. 3.0 g/kg/day
E. 4.0 g/kg/day

76. A patient with multiple small bowel resections for Crohn’s disease presents with an acute flare.
Due to his preexisting short length of bowel and current profuse diarrhea, he is suspected of having
functionally a short gut syndrome and is started on TPN. Refeeding syndrome is characterized by
which of the following electrolyte abnormalities?

A. Hyponatremia, hypokalemia, and hypercalcemia


B. Hyperphosphatemia, hypokalemia, and hypocalcemia
C. Hypokalemia, hypomagnesemia, and hypophosphatemia
D. Hypocalcemia, hyponatremia, and hypomagnesemia
E. Hyperkalemia, hypernatremia, and hypercalcemia

o Decreased serum potassium, magnesium, and phosphorus;

77. After 4 days of TPN, the patient above develops blood glucose levels greater than 300 mg/dL.
Hyperglycemia in a surgical patient receiving TPN may best be managed by:

A. Oral hypoglycemics
B. Decreasing the dextrose load and doubling the amount of fat
C. Adding regular insulin to the TPN
D. Discontinuing TPN for 2 weeks and then trying to start TPN again
E. Increasing the concentration of protein and carbohydrate calories and decreasing that of
lipids
78. A 40-year-old man undergoes gastric bypass surgery for morbid obesity. This patient should:

A. Eat only high-protein foods


B. Begin a feeding regimen with small amounts of regular foods
C. Begin with small amounts of water
D. Eat high-calorie foods six times a day
E. Eat only small amounts of high-fat foods

79. Which of the following is true when considering the nutritional status of a geriatric patient?

A. Muscle wasting can be a pathologic process that is mistaken for normal aging.
B. Liver function does not affect the selection of nutrition regimens.
C. Enteral nutrition is not an option because of slowed gut function.
D. Body mass index (BMI) is the best anthropometric measurement for determining nutritional
status in an elderly patient.
E. Laboratory tests cannot be used to evaluate nutritional status.

80. Which of the following information would not be a typical component of the Subjective Global
Assessment tool?

A. Weight changes
B. Serum albumin level
C. Changes in muscle mass
D. Dietary changes
E. Evaluation of gastrointestinal symptoms

81. The earliest sign of Hypocalcemia is?

A. Carpopedal spasms.
B. Positive chrosteks' sign.
C. Positive trousseau's sign.
D. Defective blood coagulation.
E. Tingling of fingers and circumoral region.

82. Resection of the terminal ileum is associated with malabsorption of?

A. Iron.
B. Folic acid.
C. Cholesterol.
D. Vitamin B12.
E. Calcium.

83. A 70-year-old man, 24 hours after colon resection, urine output in is 10 mL/h. Blood chemistry
analysis reveals sodium, 138 mEq/L; potassium, 6 mEq/L; chloride, 100 mEq/L; bicarbonate, 14
mEq/L. His metabolic abnormality is characterized by which of the following.

A. Abdominal distension
B. Peaked T waves
C. Narrow QRS complex
D. Cardiac arrest in systole
E. J wave or Osborne wave

o Osborne (J) wave is seen in hypothermia.


84. A 55-year-old man with Crohn’s disease had undergone resection of small bowel and
anastomosis. Ten days later, he is found to have bilious drainage of 1 L/d from the drains. He is
started on total parenteral nutrition (TPN).Four days later, his arterial blood gases (ABGs) are pH,
7.25; PO2, 98 mm Hg; and PCO2, 40 mm Hg. His anion gap is 10. The most likely cause is which of
the following?

A. Diabetic ketoacidosis
B. Renal failure
C. Hypovolemic shock
D. Small-bowel fistula
E. Uncompensated metabolic alkalosis

o This patient has metabolic acidosis with normal anion gap.


o Loss of bicarbonate e.g., small bowel fistula, result in metabolic acidosis with normal anion
gap.

85. A42-year-old man with small-bowel fistula has been receiving TPN with standard hypertonic
glucose-amino acid solution for 3 weeks. The patient is noticed to have scaly, hyperpigmented
lesions over the acral surfaces of elbows and knees, similar to enterohepatic acrodermatitis. What
is the most likely cause of the condition?

A. Copper deficiency
B. Essential fatty acid deficiency
C. Excess glucose calories
D. Hypomagnesemia
E. Zinc deficiency

Zinc deficiency Most common trace mineral deficiencies


Presented with eczematoid rash developing both diffusely and at
intertriginous areas
Copper deficiency Microcytic anemia
Chromium deficiency Glucose intolerance
Manganese deficiency Poor wound healing.
Essential fatty acids Dermatitis, diarrhea, alopecia, easy bruising, bleeding tendencies, and
deficiency delayed wound healing.
Selenium deficiency Cardiomyopathy, diffuse skeletal myalgia, loss of pigmentation, and
erythrocyte macrocytosis

86. Initial resuscitation is best done by administration of which of the following?

A. D5W
B. D5W and 0.45% normal saline
C. Ringer’s lactate solution
D. 5% plasma protein solution
E. 5% hydroxyethyl starch solution

7. A sudden onset of glucose intolerance in patients receiving total parenteral nutrition often
indicates:

A. Diabetes mellitus
B. Sepsis
C. Hypophosphatemia
D. Adrenal insufficiency
E. Zinc insufficiency
88. It is stated that enteral nutrition is safer than parenteral nutrition. Which of the following may
not be a complication of enteral nutrition?

A. Hyperosmolar, nonketotic coma.


B. Vomiting and aspiration.
C. Pneumatosis cystoides intestinalis.
D. Perforation and peritonitis.
E. Septicaemia

89. Regarding crystalloid solutions all are true EXCEPT

A. Normal saline contains 154 mmol sodium and 154 mmol of chloride
B. 3 litres of dextrose saline in a day will provide 90 mmol of sodium
C. 2 grams of potassium chloride is equal to 57 mmol of the salt
D. Hartmann's solution contains calcium and bicarbonate
E. The daily maintenance potassium requirement of a 40 Kg woman is about 40 mmol

o Molecular weight of KCL 75 g\mol 2g of kcl =27 mml

90. Hypokalemic alkalosis is commonly seen in surgical wards in?

A. Excessive absorption of alkali


B. Excessive vomiting in pyloric stenosis
C. Excessive steroid use
D. Diarrhea with dehydration

91. With regard to total body water, all of the following are true except

A. 50 to 70% of TBW is water.


B. In general the percentage of TBW that is water is higher in males than in females.
C. Lean individuals have a greater proportion of water (relative to body weight) than do obese
individuals.
D. The percentage of total body weight that is water increases with age.

92. Which of the following statements about total body water composition are correct?

A. Females and obese persons have an increased percentage of body water.


B. Increased muscle mass is associated with decreased total body water.
C. Newborn infants have the least proportion of total body water.
D. Total body water decreases steadily with age.

93. Which of the following statements about extracellular fluid are true?

A. The total extracellular fluid volume represents 40% of the body weight.
B. The plasma volume constitutes one fourth of the total extracellular fluid volume.
C. Potassium is the principal cation in extracellular fluid.
D. The protein content of the plasma produces a lower concentration of cations than in the
interstitial fluid.
E. The interstitial fluid equilibrates slowly with the other body compartments.

94. Normal anion gap acidosis is associated with:

A. ketoacidosis
B. Lactic acidosis
C. Salicylate poisoning
D. Severe diarrhea
E. Uremic acidosis

95. For a patient with a serum potassium level of 7 mEq/dL and an absent P-wave on
electrocardiogram, the initial management is:

A. Intravenous Lasix
B. Intravenous glucose/insulin
C. Kayexalate enema
D. Intravenous sodium bicarbonate
E. Intravenous calcium gluconate

96. The main fuel for colonocytes is:

A. Glutamine
B. Short-chain fatty acids
C. Alanine
D. Glucose
E. Ketones

97. A 34-year-old male has serum sodium of 114 mEq/L. Correction of hyponatremia can be done by
raising serum sodium by what amount?

A. 1 mEq/L/h
B. 3 mEq/L/h
C. 5 mEq/L/h
D. 7 mEq/L/h
E. 10 mEq/L/h

98. Which of the following statements about the differential diagnosis of hypercalcemia is correct?

A. Malignant tumors typically cause hypercalcemia by ectopic production of parathyroid


hormone (PTH).
B. The diagnosis of primary hyperparathyroidism is supported by these serum levels: calcium,
10.8; chloride, 104; bicarbonate 21; phosphorus, 2.4; elevated parathyroid hormone.
C. Familial hypocalciuric hypercalcemia is distinguished from primary hyperparathyroidism by
parathyroid imaging.
D. Although serum albumin binds calcium, the measured total calcium value is usually
unaffected in patients with severe hypoproteinemia.
E. Thiazide diuretics are a good treatment for hypercalcemia and can be given to patients with
apparent hypercalcemia of malignancy.

99. Which of the following statements about head injury and concomitant hyponatremia are true?

A. There are no primary alterations in cardiovascular signs.


B. Signs of increased intracranial pressure may be masked by the hyponatremia.
C. Oliguric renal failure is an unlikely complication.
D. Rapid correction of the hyponatremia may prevent central pontine injury.
E. This patient is best treated by restriction of water intake.

100. Calcium metabolism: All are true, except

A. Its main function is neuromuscular stability


B. Asymptomatic hypercalcemia may occur with hypoproteinemia
C. Metastatic breast cancer can be a cause of hypercalcemia
D. Calcium is given routinely in massive blood transfusion
E. Corticosteroids decrease resorption of calcium from bone
101. Crystalloid solutions: All of the following types of intravenous fluids are crystalloids, except:

A. Normal saline
B. Dextran 70
C. Hypertonic saline
D. Ringer lactate
E. 5% dextrose water

102. Hypernatremia (postoperative): All are true, except:

A. May indicate a deficit of total body water


B. May be caused by a high protein intake
C. May diagnose extracellular fluid volume depletion
D. A common cause is excessive extrarenal water loss
E. Replacement of lost water with isotonic salt solutions can produce hypernatremia

103. Interstitial fluid (ISF) compartment: is characterized by all the followings, except:

A. It is a pathway for fluids between the cells and blood


B. It contains less protein than plasma
C. Its sodium cations content is similar to that of plasma
D. The synovial fluid water is poorly exchangeable
E. The third space is normally a part of ISF

104. Acid Base Balance: Serum pH is a measure of the relative acidity or alkalinity of blood and is a
reflection of the concentration of Hydrogen ions in blood. A high concentration gives a low pH
(acid), while a low concentration gives a high pH (alkaline). What is the normal range for arterial
pH?

A. 7.02 to 7.52
B. 7.42 to 7.52
C. 7.32 to 7.42
D. 6.32 to 6.42
E. 6.22 to 6.32

105. Daily basic nutritional requirements per kilogram body weight: All are true, except:

A. Water 30-50 ml
B. Calories 30-50 kcal
C. Nitrogen 0.5-0.6 gm
D. Sodium 0.9 -1.2 mmol
E. Potassium 0.7-0.9 mmol

106. Enteral feeding is preferred over parenteral feeding for the following reasons, except:

A. It maintains the mucosal integrity of the intestine


B. Is well tolerated by most patients
C. It keeps the normal microbial flora of the intestine
D. Is indicated in high-output gastro-intestinal fistulas
E. Decreases the translocation of bacteria and toxins from intestine

107. For correction of pre-existing dehydration: All are true, except:

A. Bowel losses come from the extracellular fluid


B. Pure water losses are from the total body water
C. Protein containing fluid is lost from the plasma
D. The only hypotonic solutions in the body are saliva and sweat
E. Losses from extracellular compartment are much better tolerated than those from
intracellular compartment
108. Total parenteral nutrition (TPN): All are true, except:

A. It should be given through a central vein


B. The main source of calories has to be of carbohydrates
C. Average calorie requirement is 30-40 K Calorie per kg body weight
D. Average protein requirement is 3 grams per Kg body weight
E. Commonest metabolic complication is hyperosmolar non-ketotic hyperglycemia

109. Acute hypercalcemia. The first line of management is:

A. Fluid volume correction


B. Furosemide
C. Steroids
D. Hemodialysis
E. Mithramycin

110. All the followings are hazards of bicarbonate therapy, except:

A. Shifts the hemoglobin dissociation curve to the left


B. Paradoxical acidosis
C. Hypernatremia and hyperosmolality
D. Decreases the plasma level of ionized calcium
E. Activates the simultaneously administered catecholamines

111. Total parenteral nutrition (TPN): Deficiency of which of the following mineral occurs in patient
on TPN:

A. Chromium
B. Manganese
C. Zinc
D. All of the above
E. None of the above

112. Malabsorption, simple tests include all the following, except:

A. Microscopic examination.
B. D-xylose absorption.
C. A 72-hour stool collection for fats.
D. Small bowel x-ray series.
E. Enteroscopy

113. With regard to total body water, all of the following are true except

A. 50 to 70% of total body weight is water.


B. In general the percentage of total body weight that is water is higher in males than in
females.
C. Lean individuals have a greater proportion of water (relative to body weight) than do obese
individuals.
D. The percentage of total body weight that is water increases with age.
E. Body water is divided into extracellular (i.e. intravascular and interstitial) and intracellular
functional compartments.
114. With regards to the distribution and composition of the body fluid compartments, which of the
following statement is incorrect:

A. The majority of intracellular water is in skeletal muscle.


B. The major intracellular cation is sodium.
C. The major intracellular anions are the proteins and phosphates.
D. The major extracellular cation is sodium.
E. The major extracellular anions are chloride and bicarbonates.

115. With regard to vitamin D physiology, one is true:

A. The major non-dietary source of vitamin D is hepatic synthesis.


B. Hydroxylation of vitamin D3 results in a loss of metabolic activity.
C. Vitamin D decreases intestinal absorption of dietary calcium.
D. Vitamin D has a direct effect on bone resulting in ossification of the bone.
E. Increased level of PTH inhibits the hydroxylation of 25- hydroxycholiecalciferol in the
kidneys.

116. Regarding calcium, all true except:

A. Total plasma levels are about 2.5mmol/L (10 microgram/dl).


B. Is actively absorbed in the intestine mainly in duodenum and passively in jejunum
C. Given intravenously as gluconate.
D. Is useful in the management of hypokalemia.
E. Levels are raised in blood taken from a standing subject.

117. Causes of metabolic acidosis include all the following EXCEPT:

A. Diabetes millets
B. Vomiting
C. Starvation
D. Renal failure
E. Small bowel fistula

118. Metabolic acidosis with a normal anion gap (AG) occurs with

A. Diabetic acidosis
B. Renal failure
C. Severe diarrhea
D. Starvation

o Also, fistulas (enteric, pancreatic, or biliary), ureterosigmoidostomy,

119. All are possible causes of postoperative hyponatremia except

A. Excess infusion of normal saline intraoperatively.


B. Administration of antipsychotic medication.
C. Transient decrease in antidiuretic hormone (ADH) secretion.
D. Excess oral water intake.

120. Which of the following is an early sign of hyperkalemia?

A. Peaked T waves
B. Peaked P waves
C. Peaked (shortened) QRS complex
D. Peaked U waves
121. The next most appropriate test to order in a patient with a pH of 7.1, Pco 2 of 40, sodium of
132, potassium of 4.2, and chloride of 105 is

A. Serum bicarbonate
B. Serum magnesium
C. Serum ethanol
D. Serum salicylate

o AG = [Na] – [Cl + HCO3]

122. Which of the following is not true regarding hypertonic saline?

A. Is an arteriolar vasodilator and may increase bleeding


B. Should be avoided in closed head injury
C. Should not be used for initial resuscitation
D. Increases cerebral perfusion

o Hypertonic saline (7.5%) increase cerebral perfusion and decrease intracranial pressure, thus
decreasing brain edema.

123 .Normal saline is

A. 135 mEq NaCl/L


B. 145 mEq NaCl/L
C. 148 mEq NaCl/L
D. 154 mEq NaCl/L

124. Fluid resuscitation using albumin

A. Is associated with coagulopathy


B. Is available as 1% or 5% solutions
C. Can lead to pulmonary edema due to increased intravascular oncotic pressure
D. Decreases factor XIII

o Albumin available is available as 5% (osmolality of 300 mOsm/L) or 25% (osmolality of 1500


mOsm/L).
o Hydroxyethyl starch solutions (HES/HAES) are associated with postoperative bleeding and
associated with coagulopathy
o One liter of HES 6% solution reduces factor VIII level by 50% and will prolong PTT

125. If a patient’s serum glucose increases by 180 mg/dL, what is the increase in serum osmolality,
assuming all other laboratory values remain constant?

A. Does not change


B. 8
C. 10
D. 12

126. The free water deficit of a 70 kg man with serum sodium of 154 is

A. 0.1 L
B. 0.7 L
C. 1L
D. 3.5 L

o Water deficit = (serum sodium -140) / 140 x TBW


127. A patient with serum calcium of 6.8 and albumin of 1.2 has a corrected calcium of

A. 7.7
B. 8.0
C. 8.6
D. 9.0

128. All the following treatments for hyperkalemia reduce serum potassium except

A. Bicarbonate
B. Kayexalate
C. Glucose infusion with insulin
D. Calcium gluconate

129. An alcoholic patient with serum albumin of 3.9, K of 3.1, Mg of 2.4, Ca of 7.8, and PO4 of 3.2
receives three boluses of IV potassium and has serum potassium of 3.3. You should

A. Continue to bolus potassium until the serum level is >3.6.


B. Give MgSO4 IV.
C. Check the ionized calcium.
D. Check the BUN and creatinine.

130. A patient who has spasms in the hand when a blood pressure cuff is blown up most likely has

A. Hypercalcemia
B. Hypocalcemia
C. Hypermagnesemia
D. Hypomagnesemia

o Asymptomatic hypocalcemia may occur with hypoproteinemia (normal ionized calcium), but
symptoms can develop with alkalosis (decreased ionized calcium).
o In general, symptoms do not occur until the ionized fraction falls below 2.5 mg/dL,

130. The effective osmotic pressure between the plasma and interstitial fluid compartments is
primarily controlled by

A. Bicarbonate
B. Chloride ion
C. Potassium ion
D. Protein

131. The metabolic derangement most commonly seen in patients with profuse vomiting

A. Hypochloremic, hypokalemic metabolic alkalosis


B. Hypochloremic, hypokalemic metabolic acidosis
C. Hypochloremic, hyperkalemic metabolic alkalosis
D. Hypochloremic, hyperkalemic metabolic acidosis

132. When lactic acid is produced in response to injury, the body minimizes pH change by

A. Decreasing production of sodium bicarbonate in tissues


B. Excreting carbon dioxide through the lungs
C. Excreting lactic acid through the kidneys
D. Metabolizing the lactic acid in the liver
133. What is the best determinant of whether a patient has a metabolic acidosis versus alkalosis?

A. Arterial pH
B. Serum bicarbonate
C. Pco2
D. Serum CO2 level

134. Which of the following are NOT characteristic findings of acute renal failure and urgent
correction is mandatory?

A. BUN >100 mg/dL


B. Hyperkalemia
C. Severe acidosis
D. Uremic pericarditis
E. Uremic encephalopathy

135. Excessive administration of normal saline for fluid resuscitation can lead to what metabolic
derangement?

A. Metabolic alkalosis
B. Metabolic acidosis
C. Respiratory alkalosis
D. Respiratory acidosis

136. The first step in the management of acute hypercalcemia should be

A. Correction of deficit of extracellular fluid volume


B. Hemodialysis.
C. Administration of furosemide.
D. Administration of mithramycin.

137. Which of the following is the largest fluid compartment in the body?

A. Plasma
B. Central spinal fluid
C. Interstitial fluid
D. Intracellular fluid

138. If 1 liter of 0.9% NaCl solution is given intravenously, how much will be distributed to the
interstitial space?

A. 100 cc
B. 250 cc
C. 400 cc
D. 750 cc

139. A patient develops a high output fistula following abdominal surgery. The fluid is sent for
evaluation with the following results: Na 135 K 5 Cl 70. Which of the following is the most likely
source of the fistula?

A. Stomach
B. Small bowel
C. Pancreas
D. Biliary tract
140. A patient is admitted with a glucose of 500 and a sodium of 151. Which of the following is the
best approximation of the patient's actual serum sodium level?

A. 158
B. 151
C. 145
D. 138

141. Which of the following is the most likely diagnosis in a patient with a serum sodium of 152
mEq/L, a urine sodium concentration of >20 mEq/L, and a urine osmolality of >300 mOsm/L?

A. Syndrome of inappropriate anti-diuretic hormone (SIADH)


B. Diabetes insipidus
C. Renal tubular disease
D. Cushing's syndrome

142. Which of the following can contribute to hyperkalemia in patients with renal insufficiency?

A. Loop diuretics
B. Aspirin
C. Calcium channel blockers
D. Nonsteroidal anti-inflammatory drugs (NSAIDs)

143. Which of the following would cause decreased deep tendon reflexes?

A. Hypokalemia
B. Hypomagnesemia
C. Hypocalcemia
D. Hypoglycemia

o Hypokalemia causes decreased deep tendon reflexes.


o Hypomagnesemia and hypocalcemia cause increased deep tendon reflexes

144. Which of the following is an early ECG change seen in hyperkalemia?

A. Prolonged PR interval
B. Sine wave formation
C. Peaked T waves
D. Flattened P wave

145. What is the actual serum calcium level in a patient with an albumin of 2.0 and a serum calcium
level of 6.6?

A. 6.6
B. 7.4
C. 8.2
D. 9.9

146. Which of the following is a cause of acute hypophosphatemia?

A. Chronic ingestion of magnesium containing laxatives


B. Insulin coma
C. Refeeding syndrome
D. Rhabdomyolosis

o Rhabdomyolosis is associated with hyperkalemia and hyperphosphatemia


147. A patient presents obtunded to the ER with the following labs: Na 130 Cl 105 K 3.2 HCO3 15
Which of the following is the most likely diagnosis?

A. GI losses
B. Lactic acidosis
C. Methanol ingestion
D. Renal failure

 This is a normal anion gap acidosis.

148. Which of the following is the best choice to replace isotonic (serum) fluid loss?

A. D5 ¼ NS with 20 mEq KCl/liter


B. D5 ½ NS with 20 mEq KCl/liter
C. 3% saline solution
D. Lactated Ringer's

149. A postoperative patient with a potassium of 2.9 is given 1 mEq/kg replacement with KCl
(potassium chloride). Repeat tests after the replacement show the serum K to be 3.0. The most
likely diagnosis is

A. Hypomagnesemia
B. Hypocalcemia
C. Metabolic acidosis
D. Metabolic alkalosis

150. Hypomagnesemia clinically resembles which of the following?

A. Hypoglycemia
B. Hypokalemia
C. Hypophosphatemia
D. Hypocalcemia

151. A 25-year-old female driver of an automobile sustained severe multiple traumatic injuries. She
had an altered sensorium and required intubation. Etomidate was used for induction. A noted side
effect of this agent is associated with what electrolyte abnormality?

A. Hypomagnesemia
B. Hypokalemia
C. Hyperphosphatemia
D. Hyponatremia and hyperkalemia
E. Hypernatremia and hypokalemia

152. Which of the following conditions is associated with hypovolemic hyponatremia?

A. Cirrhosis
B. Congestive heart failure
C. Nephrotic syndrome
D. Enterocutaneous fistula
E. SIADH

153. Hypernatremia in a volume-depleted patient is best treated with what initial fluid choice?

A. Normal saline
B. 0.25% saline
C. 3% saline
D. Lactated Ringer’s
E. No fluid should be given
154. A 60-year-old cancer patient has gastric outlet obstruction and presents with refractory
vomiting. Which intravenous fluid is most appropriate for volume repletion?

A. Normal saline
B. 0.45% saline
C. 3% saline
D. Lactated Ringer’s
E. Free water

155. Routine lab work on a patient with small bowel obstruction shows refractory hypocalcemia
causing perioral numbness that is uncorrectable with calcium supplementation. What other
electrolyte level must be checked and corrected?

A. Sodium
B. Phosphate
C. Bicarbonate
D. Magnesium
E. Chloride

156. A patient with Crohn disease undergoes an ileocecectomy for perforation and subsequently
has a prolonged postoperative ileus. What is the most common electrolyte abnormality associated
with starting TPN?

A. Hypernatremia
B. Hypokalemia
C. Hypophosphatemia
D. Hypomagnesemia
E. Hypochloremia

157. The best nutrition access method for a critically ill burn patient is:

A. Nasogastric
B. Nasojejunal
C. Parenteral
D. Percutaneous gastrostomy
E. Witzel jejunostomy

158. Generalized edema results from all of the following disorders except

A. Systemic hypertension.
B. Congestive heart failure.
C. Cirrhosis.
D. Nephrotic syndrome.
E. Hyperaldosteronism

159. All the following statements about vitamin A are true except

A. Deficiency causes blindness.


B. Liver disease causes deficiency.
C. Toxicity causes liver disease.
D. Deficiency causes squamous metaplasia in glandular epithelium.
E. Toxicity is associated with increase infections.
160. Which of the following statements about vitamin K deficiency is true:

A. Deficiency causes defects in both the intrinsic and extrinsic coagulation pathways.
B. Deficiency produces defects in platelets function.
C. Deficiency is associated with strict vegetarian diet.
D. Interstitial bacteria produce enough vitamin K to prevent deficiency under normal
conditions.
E. Deficiency develops less frequently in breast fed infants than in bottle fed infants.

161. Which of the following (ECG) findings is not associated with hyperkalemia?

A. Peaked T waves
B. Prolong PR interval
C. Loss of P wave
D. Narrowing of QRS complex.
E. T waves higher than R waves in more than one level.

162. With regard to hypokalemia, which of the following statements is not true?

A. K+ and H+ are exchanged for Na+ in renal tubule.


B. Respiratory acidosis is associated with increase renal K+ loss.
C. Hypokalemia can cause decrease deep tendon reflexes.
D. Flattened T waves and a prolonged QT interval are associated with hypokalemia.
E. Intravenous K+ administration should not exceed 40 to 60 mEq/hour.

163. Which of the following clinical scenario is not associated with acute hypocalcemia?

A. Fluid resuscitation from shock.


B. Rapid infusion of blood products.
C. Improper administration of phosphates.
D. Vitamin D-deficient diets.
E. Acute pancreatic.

164. All of the following symptoms are seen with hypermagnesemia except:

A. Tachypnea
B. Depression of reflexes
C. Arrhythmias
D. Hypotension
E. Central nervous system depression

165. As the amount of total body fat increase, the total body water:

A. Increases.
B. Decrease.
C. Unchanged.
D. Depends on patient's age.
E. Depends on patient's sex.

166. One week of starvation results in a loss of approximately 100 gm of nitrogen (13.8 gm N/ day).
This is associated with protein loss of:

A. 625 gm.
B. 100 gm.
C. 1000 gm.
D. 50 gm.
E. 16 gm.
167. The highest potassium content is in:

A. Saliva.
B. Gastric juice.
C. Bile.
D. Ileal
E. Duodenal

o In saliva 26 mmol\L. Colon 30 mmol \L

168. Metabolic acidosis may be brought by

A. Loss of CO2 by increase ventilation


B. Retention of CO2 by respiratory obstruction
C. Persistent vomiting
D. Absorption of excessive amount of Na HCO3
E. Excessive exercise

169. The predominant buffering system in humans is:

A. Bicarbonate.
B. Titratable acids.
C. Ammonium (NH4+).
D. Sodium lactate
E. Phosphate

170. Calcium is maximally absorbed in which portion of the gastrointestinal tract?

A. Stomach
B. Jejunum
C. Jejunum and proximal ileum
D. Ileum
E. Ascending colon

171. The calorie-nitrogen ratio for infant should be maintained at:

A. 75:1
B. 100:1
C. 50:1
D. 150:1
E. 25:1

172 .A patient has undergone an ileal resection. Which of the following conditions would he be
least likely to develop?

A. Alopecia
B. Megaloblastic anemia
C. Nephrolithiasis
D. Cholelithiasis
E. Steatorrhea

173. The substance involved with protein synthesis

A. DNA
B. RNA
C. Adenine
D. Guanine
E. Cytosine
174. The average daily loss of water through the lungs and skin (insensible water loss) is
approximately:

A. 10 ml
B. 100ml
C. 1000ml
D. 2000ml
E. Any of these depending on circumstances

175. The concentration of which electrolyte in pancreatic secretion increases as the rate of
secretion increases?

A. Sodium
B. Potassium
C. Chloride
D. Bicarbonate
E. Calcium

176. To maintain a normal hydrogen balance, total daily excretion of H+ should equal the daily:-

A. Fixed acid production plus fixed acid ingestion.


B. HCO3- excretion
C. Pseudomonas aeruginosa.
D. S.Fecales.
E. Staph. Aureus.

177. In electrolyte disorders that occur when jejunum is used for urinary intestinal diversion, one is
not happened:

A. Hyponatremia
B. Hyperchloremia
C. Hyperkalemia
D. Azotemia
E. Metabolic acidosis

178. Regarding colloid solutions, all are true, except:

A. Human albumin has a molecular weight of 69 kDa.


B. Albumin has a half-life in the circulation of about 15 days.
C. Gelatins (e.g. Haemaccel®) are polysaccharides with a MW of about 35 kDa.
D. Dextrans reduce platelet aggregation and can induce anaphylaxis.
E. 6% Hydroxylethyl Starch (HES) is a synthetic polysaccharide derived from amylopectin.

o Dextrans Polysaccharides with MW 10-90 kDa. Reduces plasma viscosity and platelet
aggregation. 1-5% develop anaphylaxis
o Gelatins: Polypeptides with MW ~35 kDa, rapidly lost from vascular space
o Hydroxyethyl starch: Synthetic polysaccharide polymers derived from amylopectin with MW
50-450 kDa. Associated with bleeding diathesis

179. Regarding central parenteral nutrition, all are true, except:

A. Is a hypoosmolar solution.
B. Typically contains 14-16g nitrogen as L-amino acids.
C. Typically contains about 250g glucose.
D. Is associated with metabolic disturbances in about 5% of patients.
E. Can induce derangement of liver function tests.
180. In the body metabolism, 10g of protein, would produce approximately:

A. 20 kcal.
B. 41 kcal.
C. 410 kcal.
D. 4100 kcal.
E. 900 kcal.

181. All are causes of metabolic acidosis, except:

A. Untreated diabetes mellitus.


B. Duodenal Ulcer.
C. Uremia.
D. Shock.
E. Liver failure

182. A child weighs 15 Kg, his fluid requirement is:

A. 50 ml per hour.
B. 50 ml per kg per hour.
C. 1000 ml per 24 hours.
D. 10 ml per kg per hour.
E. 150 ml per hour.

183. One of the followings is not a cause of hypokalemia?

A. Inadequate oral intake.


B. Potassium-free intravenous fluids.
C. High output nasogastric tube.
D. Blood transfusion.
E. Massive vomiting and diarrhea.

184. The most common cause of hypercalcemia in hospitalized patients is?

A. Malignancy.
B. Over oral intake.
C. Bone fracture.
D. Blood transfusion.
E. Parathyroid adenoma.

185 .One of the following is not a cause of dilutional hyponatremia?

A. Excessive oral water intake.


B. Iatrogenic (intravenous) excess of free water administration.
C. Postoperative increase of antidiuretic hormone secretion.
D. Decreased oral intake of sodium.
E. Some drugs which cause water retention.

186. The following cause hypercalcemia EXCEPT

A. Sarcoidosis
B. Primary hyperparathyroidism
C. Acute pancreatitis
D. Metastatic bronchial carcinoma
E. Milk-Alkali syndrome
187. The following are complications of parenteral nutrition

A. Hyperglycemia
B. Hypoglycemia
C. Hypokalemia
D. Hyperkalemia
E. All of the above

188. The following statements concern potassium are true, except

A. 30% of total body potassium is located in the extracellular compartment.


B. Aldosterone increase active potassium secretion in the distal convoluted tubule.
C. Hypokalemia directly stimulate aldosterone secretion.
D. Insulin cause potassium to enter the cell.
E. Hyperkalemia will occur if the activity of the membrane-bound Na/K pump is impaired

189. Hydrogen ion (H+) homeostasis which is true?

A. Proteins are the principle buffer in urine.


B. Hydrogen ions are secreted into the renal tubular lumen in exchange for potassium ions, so
that they combine with filtered bicarbonate.
C. Bicarbonate is the most important buffer in the extracellular fluid.
D. Ammonia ions (NH4+) can cross the membrane of renal tubular cell.
E. The hydrogen ion concentration in arterial blood is inversely proportional to pCO2.

190. Which is false regarding assessment of nutritional status?

A. Height (meters) divided by weight (kg) squared ( H/W2)


B. Measurement of urinary urea over 24 hours.
C. Bioelectrical impedance.
D. Measurement of handgrip strength.
E. Measurement of serum transferring level.

191. The following are true regarding enteral feeding except.

A. It is the route of choice if the gastrointestinal tract is intact and functioning.


B. Element diets contain whole protein.
C. Polymeric diets provide energy in the form of complex carbohydrate and fat.
D. It is contraindicated in patients with intestinal obstruction.
E. Feeding jejunostomies reduce the risk of pulmonary aspiration.

192. The following statement regarding intravenous fluid and their component is true:

A. Dextrose saline is made up of 4% glucose and 0.9% sodium chloride.


B. Hartmann’s solution contain 131mmole/liter of sodium and 5mmole/ liter potassium
C. 3 liters of dextrose saline provide similar of sodium as 1 liter of normal saline.
D. Normal saline contain 120 mmole/ liter of chloride ions.
E. 5% DEXTROSE provide 250kcal of energy.

193. How much of the daily insensible water loss is due to loss in stool?

A. 200 ml
B. 400 ml
C. 600 ml
D. 800 ml
E. 1000 ml

194. Regarding enteral feeding which one of the following is FALSE:

A. Fine bore NG tube is ideal for a deeply comatose patient


B. PEG tube is nowadays inserted by endoscopy with local anesthesia
C. 30% of patients have diarrhea
D. PEG tube may be complicated by persistent gastric fistula

195. All of the following about ringer lactate are true, except:

A. Ca = 2.8
B. K= 4
C. Lactate = 52
D. Na = 130
E. Cl=109

196. Which of the following statement is true concerning parental electrolyte solutions?

A. Lactated Ringer solution contains physiologic concentrations of all important electrolytes.


B. Sucrose is added to hypotonic saline solution to increase their tonicity.
C. Negligible amount of exogenously administered albumin end up in the extravascular space.
D. Normal saline solution provides excessive sodium and chloride, and the excess can cause
body sodium overload.
E. Hypernatremia can occur with extended use of lactated Ringer solution.

197. Which of the following statements is false concerning the derangement of metabolic acidosis.

A. An important acid in the body is sulfuric acid.


B. Excessive loss of bicarbonate can occur with intestinal or pancreatic fistula.
C. Ketoacidosis can occur with either hyperglycemia or hypoglycemia.
D. Lactic acidosis is present when serum lactate concentration is greater than 2 meq/L.
E. Lactic acidosis can be associated with ethanol toxicity.

198. Ingested calcium is primarily absorbed in the:

A. Stomach.
B. Jejunum.
C. Proximal ileum.
D. Distal ileum.
E. Doudenum.

199. Clinical manifestations of acute metabolic acidosis include which of the following?

A. Increased cardiac contractility


B. Decreased catecholamine secretion
C. Peripheral arteriolar dilatation
D. Shift of the oxygen-hemoglobin dissociation curve to the left.
E. Central venous dilatation.
200. Which of the following statements is true concerning method of nutritional support?

A. Optimal results for enteral feeding are achieved with approximately one half of calories
supplied as carbohydrate and one half as protein.
B. Diarrhea is the most common complication of enteral feeding and is caused by high
osmolarity of the fat components.
C. The hyperosmolar nature of parenteral fat solution require central venous administration
D. Approximately 25% to 50% of calories should be provided as fat emulsion to patients
receiving total parental nutrition.
E. Most formula for enteral feeding range from 4.0 to 8.0 cal/ml, and include 20% protein.

201. Which of the following statements is true concerning abnormalities in serum level of sodium

A. The most cause of hyponatremia is a deficit in total body sodium


B. Hyponatremia can occur in situations of excessive solute.
C. Most surgical patients with hyponatremia are best treated with water administration
D. Hyponatremia should be rapidly corrected with free water administration.
E. Hypernatremia is quite more common than hyponatremia in surgical patients that need
urgent and rapid correction.

202. Which of the following statements is true concerning alteration in serum level of magnesium?

A. Renal failure is the primary cause of hypomagnesemia


B. Hypomagnesemia is common after acute pancreatic.
C. Symptoms of hypermagnesemia can mimic symptoms of hypocalcemia
D. Intravenous administration of magnesium sulfate usually is the most efficient method of
correction magnesium deficiency.
E. Low magnesium intake is the commonest cause of Hypomagnesemia.

203. Which of the following is true concerning the clinical presentation and management of severe
metabolic alkalosis?

A. In most cases, clinical signs are obvious.


B. Correction of potassium and volume depletion correct most cases of metabolic alkalosis.
C. Acetazolamide inhibits renal excretion of bicarbonate.
D. Acid replacement should be provided at a molar equivalent basis for excess serum
bicarbonate.
E. Acid should not be administered to titrate the excess extracellular bicarbonate.

204. All of the following as used for nutritional status assessment except:

A. Upper Arm Circumference


B. Biceps skin fold thickness
C. Albumin
D. Lymphocyte count
E. Urinary Creatinine excretion

o It is triceps not biceps

205. In a normal adult woman weighing 75kg the extracellular fluid:

A. Contains no protein
B. Forms a greater proportion of the total body weight in the obese than in the lean woman
C. Has a total volume of 12-15 litres 
D. Has a sodium concentration of 125-135mmol/l
E. Is isotonic throughout the body
206. The most common cause of hypercalcemia in hospitalized patients is:

A. Primary hyperparathyroidism
B. Metastatic carcinoma
C. Sarcoidosis
D. Immobility
E. Milk alkali syndrome

207. Magnesium:

A. Deficiency can be accurately diagnosed with serum level measurement


B. Hypomagnesemia is associated with hyperkalemia
C. Hypomagnesemia is associated with neuromuscular excitability (hyperactively)
D. Hypomagnesemia is a complication of renal failure
E. Is a major extracellular cation

208. Which of the following statements about extracellular fluid is true?

A. The total extracellular fluid volume represents 40% of the body weight.
B. The plasma volume constitutes one fourth of the total extracellular fluid volume.
C. Potassium is the principal cation in extracellular fluid.
D. The protein content of the plasma produces a lower concentration of cations than in the
interstitial fluid.

209. The interstitial fluid equilibrates slowly with the other body compartments water?

A. Osmoreceptors and baroreceptors work equally to control fluid balance during normal
conditions
B. The cardiac atrium regulates volume by means of natriuretic peptides
C. The kidney is the primary effector organ in controlling water balance
D. The conversion of angiotensin I to angiotensin II is dependent on the amount of the enzyme,
renin, available
E. Nitric oxide plays a number of important roles in regulation of renal hemodynamics

210. In metabolic acidosis:

A. There is positive base excess


B. HCO3 is the main intracellular buffer
C. Proteins and phosphates are the main extracellular buffers
D. Compensation occur by increase in alveolar ventilation
E. Bicarbonate infusion is the mainstay of treatment

o HCO3 Bicarbonate is the main extracellular buffer

211. Recognized causes of hypokalemia include all except:

A. Villous adenoma of rectum


B. Captopril therapy
C. Conn's syndrome
D. Pyloric stenosis
E. respiratory alkalosis
212. All are causes of hyperkalemia except:

A. Dopamine infusion
B. Catabolic state
C. Suxamethonium chloride
D. Burns
E. Diabetic ketoacidosis

213. Metabolic alkalosis is seen in:

A. Pancreatic fistula
B. Aspirin poisoning
C. Protracted vomiting
D. Hyperventilation
E. Acute renal failure

214. Complications of TPN include all except:

A. Hypocalcemia
B. Hypercalcemia
C. Hypokalemia
D. Hypophosphatemia
E. Hyperurecemia

215. Enteral nutrition. All true, except:

A. May cause bacterial overgrowth in the gut


B. Reduces stress ulceration
C. Is less expensive than the parenteral route
D. Decreases the incidence of cholestasis
E. May be commenced immediately after surgery

216. A 5% dextrose infusion:

A. Is hypotonic on administration
B. Remains in intravascular space for at least one hour
C. Is a good mode of resuscitation in the shocked patient
D. May give rise to type I respiratory failure
E. Contains 60 Kcal(~251 J)/L

217. Hyponatremia can occur in all, except:

A. Burns
B. Fever
C. Intestinal obstruction
D. Renal failure
E. Following TURP

o Pyrexia leads to water loss and thus Hypernatremia.

218. Hypokalemia:

A. Produces peaked T-wave on ECG


B. Causes the heart to arrest in diastole
C. Is seen in metabolic acidosis
D. May occur following extensive muscle trauma
E. May be masked by severe extracellular fluid volume deficit
219. A severely traumatized patient who has been receiving prolonged parenteral alimentation
develops diarrhea, mental status changes, alopecia, and perioral and periorbital dermatitis
Administration of which of the following trace elements is most likely to reverse these
complications?

A. Iodine
B. Zinc
C. Selenium
D. Silicon
E. Tin

220. Which statement is false regarding hypercalcemia?

A. Calcitonin is a satisfactory long-term therapy for hypercalcemia.


B. The symptom of hypercalcemia may mimic symptom of hyperglycemia (polyuria, polydipsia)
C. Metastatic breast cancer is a usual cause of hypercalcemia
D. Severely hypercalcemic patients exhibit the signs of extracellular fluid volume deficit.
E. Urinary calcium excretion may be increased by vigorous volume repletion.

o Treatment by IVF 4-6 L 0.9% with bisphosphorate


o Prednisone in some cases (MM, sarcoidosis, vitamin D deficiency)
o Calcitonin is a short-term therapy for hypercalcemia not commonly use

221. In sodium homeostasis which is true?

A. The most common cause of hyponatremia is deficit in total body sodium


B. Hyponatremia is usually the result of excessive sodium administration.
C. Most patients with hyponatremia are best treated by free-water restriction.
D. Cardiovascular systems are the predominant symptoms of hypernatremia.
E. Hypernatremia should be rapidly corrected with free-water administration.

222. The most reliable measurement for diagnosing malnutrition is a

A. Serum albumin of 2.6 g/mL


B. Ratio of exchangeable sodium (Na) to exchangeable potassium (K) of 0.9
C. Respiratory quotient on indirect calorimetry of 1.0
D. Daily calorie intake that meets < 40% of total energy needs
E. Loss of 7% of total body weight

223. The amount of glucose needed to prevent muscle catabolism in simple starvation:

A. 50 g
B. 100 g
C. 150 g
D. 200 g
E. 250 g

224. Which of the following is consistent with syndrome of inap propriate antidiuretic
hormone (SIADH)?

A. Hypovolemia
B. I ncreased urine sodium
C. Hypernatremia
D. Plasma hyperosmolarity
E. E xcessive diuresis
225. Normal anion gap acidosis is associated with:

A. ketoacidosis
B. Lactic acidosis
C. Salicylate poisoning
D. Severe diarrhea
E. Uremic acidosis

226. Phosphorus:

A. Is a major extracellular anion


B. Is passively absorbed from the gastrointestinal tract
C. Deficiency may result in insulin resistance
D. Deficiency is rare in hospitalized patients

227. Hypermagnesemia is a complication of:

A. Extensive burns
B. Acute pancreatitis
C. Oliguric renal failure
D. Resection of the terminal ileum
E. Diuretic therapy

228. The most common cause of hypercalcemic crisis is:

A. Sarcoidosis
B. Primary hyperparathyroidism
C. Secondary hyperparathyroidism
D. Renal faliure
E. Malignancy

229. Dry, scaly, pruritic rash on the trunk and extremities of a patient receiving total
parenteral nutrition is caused by:

A. Zinc deficiency
B. Vitamin A deficiency
C. Vitamin C deficiency
D. Free fatty acid deficiency

230. The most effective method of treating hyperkalemia is:

A. Intravenous calcium gluconate


B. Intravenous sodium bicarbonate
C. Hemodialysis
D. cation-exchange resin
E. Intravenous glucose-insulin

231. A patient receiving 1,800 cal/day in total parenteral nutrition will require:

A. 125 g protein/day
B. 150 g protein/day
C. 200 g protein/day
D. 250 g protein/day
E. 300 g protein/day
232. Appearance of a U-wave on electrocardiogram occurs in:

A. Hyperkalemia
B. Hypokalemia
C. Hypermagnesemia
D. Hypomagnesemia
E. Hypercalcemia

233. Headache, vomiting, and seizure may develop with rapid correction of:

A. Hyponatremia
B. Hypernatremia
C. Hypokalemia
D. Hyperkalemia
E. Hypercalcemia

o Rapid correction of hyponatremia can be associated with irreversible central nervous system
injury (central pontine and extrapontine myelinolysis). patients may present with the
neurological signs and symptoms of hyponatraemic encephalopathy such as nausea and
vomiting, confusion, headache and seizures

o Too rapid correction of hypernatremia may lead to cerebral edema (present with
nausea, vomiting, blurred vision, faintness, and in severe cases seizures

234. The respiratory quotient in prolonged starvation is:

A. 1.0
B. 1.2
C. 0.8
D. 0.7
E. 1.5

235. A respiratory quotient of 1.2 indicates:

A. Lipogenesis
B. ketogenesis
C. Pure fat utilization
D. Carbohydrates are the source of fuel
E. Proteins are the source of fuel

236. The most abundant amino acid in the body is:

A. Alanine
B. Valine
C. Leucine
D. Glutamine
E. Arginine

237. Maximum efficiency of glucose use in total parenteral nutrition occurs at the infusion
rate of:

A. 4 mg/kg/min
B. 5 mg/kg/min
C. 6 mg/kg/min
D. 7 mg/kg/min
E. 9 mg/kg/min
238. Eczematoid rash at intertriginous areas with prolonged total parenteral nutrition is
caused by:

A. Zinc deficiency
B. Fatty acid deficiency
C. Copper deficiency
D. Magnesium deficiency
E. Niacin deficiency

239. Alopecia with prolonged total parenteral nutrition may be caused by:

A. Zinc deficiency
B. Magnesium deficiency
C. Vitamin A intoxication
D. Essential fatty acids deficiency
E. Selenium deficiency

240. The initial step in management of a hypercalcemic crisis is intravenous:

A. Steroids
B. Calcitonin
C. Saline
D. Furosemide
E. Mithramycin

241. Refeeding syndrome is most commonly related to:

A. Hyponatremia
B. Hypocalcemia
C. Hypophosphatemia
D. Hypokalemia
E. hyperphosphatemia

242. The most accurate measure of adequacy of nutritional support is:

A. Serum albumin level


B. Body weight
C. Triceps skinfold measurement
D. Serum pre-albumin level

243. The most reliable means of preoperative nutritional assessment is:

A. Clinical history of weight loss


B. Serum albumin level
C. Impaired cell-mediated immunity
D. Triceps skinfold measurement

244. The most common cause of hypophosphatemia in hospitalized patients is:

A. Renal failure
B. Sepsis
C. Glucose overload (osmotic diuresis)
D. Diarrhea
245. Optimum calorie/nitrogen ratio for protein synthesis is:

A. 25-50 : 1
B. 50-75 : 1
C. 75-100 : 1
D. 100-150 : 1

246. A physiologic change of aging is:

A. Increased total body water


B. Decreased antidiuretic hormone secretion (ADH)
C. Decreased aldosterone secretion
D. Exaggerated thirst response
E. Decreased atrial natriuretic peptide (ANP)

247. In the elderly:

A. There is decreased insensible water loss


B. There is increased antidiuretic hormone response
C. There is increased volume of distribution of water-soluble medications
D. There is decreased volume of distribution of lipid-soluble medications

248. Hypophosphatemia is associated with:

A. Increased hemoglobin p50, respiratory failure, and encephalopathy


B. Decreased hemoglobin p50, hemolysis, and respiratory failure
C. Respiratory failure, hypothyroidism, and hemolysis
D. Ataxia, cardiomyopathy, and hypothyroidism

249. Most renal absorption of sodium takes place in:

A. The proximal tubules


B. The loop of Henle
C. The distal tubules
D. The collecting ducts

250. In the absence of sepsis, glucose intolerance with total parenteral nutrition may indicate:

A. Copper deficiency
B. Zinc deficiency
C. Magnesium deficiency
D. Chromium deficiency

251. The greatest amount of maintenance intravenous fluids is required for:

A. 21-year-old male athlete


B. 55-year-old obese male office worker
C. 21-year-old housewife
D. 75-year-old female with recent weight loss

252. In a normal adult woman weighing 75kg the extracellular fluid:

A. Contains no protein
B. Forms a greater proportion of the total body weight in the obese than in the lean woman
C. Has a total volume of 12-15 liters
D. Has a sodium concentration of 125-135mmol/l 
E. Is isotonic throughout the body

o Total body water is approximately 37.5 liters (0.5x70), of which 1/3 is ECF (13L) and 2/3 (22L)
intracellular fluid.
o In the obese, ECF is relatively contracted.
o Normal sodium concentration is approximately 135-145 mmol/l.
o ECF is composed of intravascular fluid and extravascular fluid.
o Both contain plasma proteins.

253. Compared with intracellular fluid, extracellular fluid has which of the following?

A. Greater osmolarity
B. Higher potassium ion concentration
C. Higher protein concentration
D. Lower chloride ion concentration 
E. Lower hydrogen ion concentration 

o Extracellular fluid has a lower osmolarity compared to intracellular fluid.


o Intracellular fluid has a protein concentration of 55 mmol/L, whilst that of extracellular fluid is 16
mmol/L.
o Intracellular fluid has a low [Cl-] whilst the extracellular fluid has a high [Cl-] with 102 mmol/L in
plasma and 114 mmol/L in the interstitial fluid.
o Intracellular fluid has a higher [H+] than extracellular fluid due to cellular metabolism.
o The intracellular [K+] is 160 mmol/L whilst that of the extracellular fluid is about 4 mmol/L

254. Which of the following is true of hypernatremia?

A. Commonly causes arrhythmias


B. May be associated with intra-abdominal hemorrhage
C. May be caused by trans-urethral resection of prostate (TURP) syndrome
D. May cause seizures 
E. Sodium levels should be corrected rapidly

o Hypernatremia patients are usually dehydrated, and the effects of hypernatremia are mainly on
the central nervous system (seizures) and muscle function.
o Cerebral dehydration with rupture of the vessels and intracranial hemorrhage may occur.
o TURP syndrome is usually associated with hyponatremia.
o Rapid correction of the serum sodium may lead to cerebral edema and convulsions, and full
correction should take at least 48 hours.

255. Which of the following is true of hypocalcaemia?

A. Associated with loss of appetite


B. Causes a shortened QT interval on the electrocardiogram
C. May be caused by pancreatitis
D. May be caused by thyrotoxicosis 
E. May cause muscle weakness 

256. You are arranging pre-operative investigations for a patient who needs a hemiarthroplasty
after sustaining a fractured neck of femur. The patient smokes and is known to have COPD. Arterial
blood gases show raised CO2 levels, reduced O2 levels, a reduced pH, and a raised bicarbonate.
What is the explanation?

A. Metabolic acidosis
B. Metabolic alkalosis
C. Partially compensated respiratory acidosis 
D. Uncompensated respiratory acidosis
E. Uncompensated respiratory alkalosis

o The reduced pH and elevated CO2 levels indicate a respiratory acidosis.


o The plasma bicarbonate concentration increases to compensate for the increased hydrogen ion
concentration. The pH is not returned to normal so there is only partial compensation.
o Respiratory acidosis is typically caused by a reduction in ventilation or ventilation/perfusion
mismatch. This can occur with chronic obstructive pulmonary disease (COPD), asthma and chest
wall injuries.

257. A middle-aged lady who is being investigated for abdominal pain 'accidentally' has a
nutritional profile taken instead of basic electrolyte estimation and this reveals an elevated
magnesium level. Her only medical history is of chronic constipation for which she has been self-
medicating with Epsom salts. What is the likely cause of her abnormal magnesium?

A. Acute pancreatitis
B. Bowel preparation
C. Diarrhea
D. Laxatives 
E. Malabsorption

o Long term treatment with Epsom salts (magnesium sulphate), a popular over the counter
preparation, may lead to hypermagnesaemia.
o The other four options, which are not uncommon scenarios in general surgical practice, all lead to
lower than desired magnesium levels.

258. Regarding central parenteral nutrition, all are true, except:

A. Is a hyper-osmolar solution.
B. Typically contains 14-16g nitrogen as D-amino acids.
C. Typically contains about 250g glucose.
D. Is associated with metabolic disturbances in about 5% of patients.
E. Can induce derangement of liver function tests.

259. Regarding postoperative nutritional support

A. Early enteral feeding is able to maintain the gut mucosal barrier


B. Enteral feeding should never be considered in the early postoperative period following
trauma or gastrointestinal surgery
C. Parenteral nutrition should always be given through a central line
D. An enterocutaneous fistula an contrindication for parenteral nutrition
E. Jaundice is a recognised complication of enteral nutrition

260. Which of the following treatments is not recommended in the treatment of hypercalcemia:

A. Thiazide diuretics
B. Corticosteroids
C. Bisphosphonates
D. Normal saline
E. Loop diuretics

261. A complication that enteral and parenteral feeding have in common is:

A. Increase incidence of sepsis


B. Intestinal villous atrophy
C. Elevated liver transaminases
D. Hyperosmolar non-ketotic
E. Diarrhea
262 .Which of the following statements about total body water composition one incorrect?

A. Females and obese persons have a decreased percentage of body water.


B. Increased muscle mass is associated with increased total body water.
C. Newborn infants have the greatest proportion of total body water.
D. Total body water decreases steadily with age.
E. Any person's percentage of body water is subject to wide physiologic variation.

263. Which of the following statements are true of a patient with hyperglycemia and
hyponatremia?

A. The sodium concentration must be corrected by 5 mEq per 100 mg. per 100 ml. elevation in
blood glucose.
B. With normal renal function, this patient is likely to be volume overloaded.
C. Proper fluid therapy would be unlikely to include potassium administration.
D. Insulin administration will increase the potassium content of cells.
E. Early in treatment adequate urine output is a reliable measure of adequate volume
resuscitation.

o Excess serum glucose acts as an osmotic diuretic, producing increased urine flow, which can
lead to volume depletion.
o Insulin therapy and the correction of the patient's associated acidosis produce movement of
potassium ions into the intracellular compartment.

264. Calculation of plasma osmolality includes each of the following factors EXCEPT:

A. Sodium.
B. Creatinine.
C. Potassium.
D. Glucose.
E. B.U.N.

265 .A non-anion gap metabolic acidosis is associated with:

A. Diabetic ketoacidosis
B. Hemorrhagic shock
C. Excessive sodium chloride administration
D. Uremia
E. Ingestion of ethylene glycol

266. Correct statements regarding lactated Ringer’s solution include which of the following?

A. It contains a higher concentration of sodium ions than does plasma


B. It is most appropriate for replacement of nasogastric tube losses
C. Is the most commonly used fluid for maintenance during anesthesia
D. It has a pH of more than 7.0.
E. It may induce a significant metabolic acidosis

267. Hyponatremia can occur in all EXCEPT:

A. Burns
B. Fever
C. Intestinal obstruction
D. Renal failure
E. Following TURP
268. Recognized causes of hypokalemia include all EXCEPT:

A. Villous adenoma of rectum


B. Captopril therapy
C. Conn's syndrome
D. Pyloric stenosis
E. Respiratory alkalosis

269. A complication that enteral and parenteral feeding have in common is:

A. Increased incidence of sepsis


B. Intestinal villous atrophy
C. Elevated liver transaminases
D. Hyperosmolar nonketotic coma
E. Diarrhea

270. All of the following are ECG features of hyperkalemia, except:

A. Sine wave pattern.


B. Loss of p waves.
C. Prolonged PR interval.
D. Peaked T wave.
E. Prolonged QT interval.

271. Most of the dietary iron is absorbed in the:

A. Stomach.
B. Upper part of small bowel.
C. Lower part of small bowel.
D. Proximal colon.
E. Distal colon.

272. Calcium reabsorption in the distal renal tubule is mediated primarily by:

A. Parathyroid hormone.
B. Vitamin D.
C. Aldosterone.
D. Hypocalcaemia.
E. Urinary sodium.

273. Hypocalcaemia associated with malignancy is most often mediated by:

A. Interleukin-6.
B. Calcitonin.
C. Parathyroid hormone related protein.
D. TNF.
E. Parathyroid hormone.

274. Ventilated patients requiring nutritional support should receive low carbohydrate and high fat
ratio nutrition for which one of following reasons:

A. High levels of carbohydrate increase co2 production.


B. High levels of carbohydrate increase water requirement.
C. High levels of carbohydrate increase fluid retention.
D. High levels of carbohydrate delay recovery from surgery.
E. High levels of carbohydrate cause re-feeding syndrome.

275. Most commercial enteral feed contains:

A. 10 Kcal /ml energy.


B. 1 Kcal /ml energy
C. 5 Kcal /ml energy.
D. 0.5 Kcal /ml energy.
E. 2.5 Kcal /ml energy.

276. Tumor lysis syndrome is associated with all of the following laboratory features, except:

A. Hyperuricaemia.
B. Hypercalcaemia.
C. Hyperkalemia.
D. Hyperphosphataemia.
E. Hyperuricosuria.

277. How much of exogenous glucose needed to prevent muscle break down in simple starvation:

A. 50 gram.
B. 120 gram.
C. 100 gram.
D. 150 gram
E. 80 gram.

278. One of the following factors stimulates the cellular uptake of potassium:

A. Antidiuretic hormone.
B. Glucagon.
C. Dopamine.
D. Aldosterone.
E. Growth hormone.

o High Insulin levels stimulate the cellular uptake of potassium


o Epinephrine: beta2 active catecholamine directly stimulate the cellular uptake of potassium

279. Each 100 gram glucose above the normal level will decrease the serum sodium level by:

A. 3 meq\L.
B. 2.6 meq\L.
C. 0.6 meq\L.
D. 1.6 meq\L.
E. 1.5meq\L.

280. One of the followings is not manifestation of hypernatermia:

A. Red swollen tongue.


B. Decrease in salivation.
C. Fever.
D. Oligouria.
E. Increase in intracranial pressure.

281. The sodium concentration of dextrose saline is:

A. 142 mmol\L.
B. 154 mmol\L.
C. 130 mmol\L.
D. 10 mmol\L.
E. 30 mmol\L.
282. Effects of hypokalemia:

A. Short PR interval
B. Ventricular extrasystoles
C. Elevated ST segments
D. Long QRS interval
E. Long QT interval

283. The most important buffer system in the intracellular fluid compartment (ICF) is?

A. Chloride buffer system


B. Protein buffer system
C. Phosphate buffer system
D. Bicarbonate buffer system
E. Sulfate buffer system

284. Which of the following is NOT the correct daily requirement for an average 70- Kg male?

A. 150mmol sodium
B. 2500 ml water
C. 70 mmol calcium
D. 70 mmol potassium
E. 70 mmol chloride

285. A patient has a total a serum calcium of 7.7, and he measured serum albumin is 2.5, what is
the corrected total calcium level?

A. 8 mg/dl
B. 9 mg/dl
C. 7.4 mg/dl
D. 8.9 mg/dl
E. 8.4 mg/dl

286. The majority of ingested iron is absorbed from?

A. Proximal ileum
B. Proximal jejunum
C. Distal ileum
D. Distal jejunum
E. Duodenum

287. One of the followings is not manifestation of hypernatermia:

A. Increase in intracranial pressure.


B. Red swollen tongue.
C. Decrease in salivation.
D. Fever.
E. Oligouria.

288. With regards to the distribution and composition of the body fluid compartments, which of the
following statement is incorrect:

A. The major extracellular cation is sodium.


B. The major extracellular anions are chloride and bicarbonates.
C. The major intracellular anions are the proteins and phosphates.
D. The majority of intracellular water is in skeletal muscle.
E. The major intracellular cation is sodium.

289. In the hypokalemia condition all of the following are true, except:

A. Causes ST segment depression on ECG.


B. Causes mental depression.
C. Precipitates digoxin toxicity.
D. May precipitate muscle paralysis.
E. Occurs in untreated hyperosmolar non-ketotic diabetic coma.

290. In metabolic acidosis:

A. There is positive base excess


B. HCO3 is the main intracellular buffer
C. Proteins and phosphates are the main extracellular buffers
D. Compensation occur by increase in alveolar ventilation
E. Bicarbonate infusion is the mainstay of treatment

291. All are causes of hyperkalemia EXCEPT:

A. Dopamine infusion
B. Catabolic state
C. suxamethonium chloride
D. Burn
E. Diabetic ketoacidosis

292. Metabolic alkalosis is seen in:

A. Pancreatic fistula
B. Aspirin poisoning
C. Protracted vomiting
D. Hyperventilation
E. Acute renal failure

293. Complications of TPN include all EXCEPT:

A. Hypocalcemia
B. Hypercalcemia
C. Hypokalemia
D. Hypophosphatemia
E. Hyperurecemia

294. Enteral nutrition, all true EXCEPT:

A. May cause bacterial overgrowth in the gut


B. Reduces stress ulceration
C. Is less expensive than the parenteral route
D. Decreases the incidence of cholestasis
E. May be commenced immediately after surgery

295. A 5% dextrose infusion:

A. Is hypotonic on administration
B. Remains in intravascular space for at least one hour
C. Is a good mode of resuscitation in the shocked patient
D. May give rise to type I respiratory failure
E. Contains 60 Kcal(~251 J)/L
296. Which of the following statement(s) is/are true concerning the initial fluid resuscitation of a
burn patient?

A. Rigid adherence to the Modified Brooke formula is advised


B. In general, children require less fluid than that predicted by standard formulae
C. Patients with inhalation injuries require less fluid than predicted by standard formulae
D. Dextrose should not be given as the primary resuscitative fluid for any age group
E. Most resuscitative formulae withhold colloid solutions until 24 hours post-injury

297. A patient who has been receiving long-term total parenteral nutrition (TPN) develops the
eczematoid rash on his face, trunk, and intertriginous areas. Which of the following should be
added to the TPN solution to correct this condition?

A. Zinc.
B. Essential fatty acids.
C. Copper.
D. Vitamin K.
E. Chromium.

298. Tumor lysis syndrome is associated with all of the following laboratory features, except:

A. Hyperuricaemia.
B. Hypercalcaemia.
C. Hyperkalemia.
D. Hyperphosphataemia.
E. Hyperuricosuria.

299. Which of the following statements regarding fluid resuscitation is true?

A. Colloids are the preferred resuscitation fluid.


B. Hydroxyethyl starch solutions are associated with increased rates of renal failure.
C. Crystalloids are the preferred resuscitation fluids.
D. Normal saline induces a metabolic alkalosis.
E. Albumin is safe to use in traumatic brain injury.

300. A 65-year-old man has a pancreatic fistula that has been draining 600–800 mL/day for the last
5 days after a pancreaticoduodenectomy 2 weeks ago. His heart rate is 115 bpm, and his urine
output is 20 mL/hour. During rounds, he suffers a grand mal seizure. A stat serum Na is 115 mEq/L,
and his serum glucose is 200 mg/dL. Which of the following statements is incorrect?

A. Hyponatremia can occur with hypotonic or hypertonic osmotic states.


B. Hypertonic saline is indicated.
C. A urine Na of 120 mEq/L is consistent with his clinical picture.
D. Hyponatremia can occur with any volume state (hypovolemia, euvolemia, or hypervolemia).
E. Octreotide may reduce his fistula output.

You might also like